Cars passage practice

अब Quizwiz के साथ अपने होमवर्क और परीक्षाओं को एस करें!

One thing we can almost all agree on is that not all cultures are alike. Clearly, there are enormous differences between the cultural orientations of western societies, for example, and the perspectives of eastern civilizations. Beyond that obvious duality, among peoples who share a more or less common historical tradition many subtle and not so subtle distinctions set the culture of one nation apart from that of another. Even within a single nation, it is not unusual to find bitterly divisive "cultural wars" raging throughout its spiritual, artistic and intellectual life. Of course, much of the cultural gap between east and west was narrowed in the first half of the 20th century by the infiltration of eastern political thought by an essentially western philosophy. In China, for instance, Marxism-Leninism had no native roots of any kind, and was imported wholesale from the neighboring behemoth to the north. Although some argue that Russia is partly an Asiatic nation, and that therefore communism can claim an eastern origin, historically that economic and political philosophy originated in England and Germany, two of the most stolidly western nations on Earth. Among the so-called "westernized" societies in Asia who have adopted a version of "free-market democracy," modernization has had an undeniably homogenizing effect, even though many purely eastern—and national—traditions remain powerful. As trade and technology in the modern era have brought the ends of the Earth closer together, a healthy dose of eastern philosophies and perspectives has infused the traditionally open western societies. These Asian influences have proven especially irresistible to artists and spiritualists. The alleged cultural differences among western societies have long been the source of nasty anecdote and sarcastic humor. Historically, the British, for example, have caricatured the French as being affected and dandified, while the French have stereotyped the British as being haughty and stiff. Perhaps when their emigres came to the western hemisphere and discovered the advantages of living together and learning from each other, most of those tensions among European cultures smoothed over. (No one familiar with the descendants of early French immigrants, for example, would call a Louisiana shrimper a dandy, nor accuse a logger in Quebec of pretension. Apart from the questions of individual honor that might be aroused, such characterizations are both ludicrous and laughable.) Nevertheless, there do remain cultural distinctions among the western nations. Europeans tend to view the United States as a nation without a culture other than the worship of the almighty dollar. And while it is true that the lure of the dollar has also, over time, entranced the cultures of Europe, it is likely also true that European societies, generally speaking, accord a higher priority to the arts than does the United States. Even though much has been written in the past decade extolling the vibrant "popular culture" of the United States, and elevating it to a level previously reserved for the great masterworks of art, it is still probably true that the most famous cultural artifacts exported by America are the golden arches of McDonald's. Within the United States, though, a real confrontation of values and cultural representation has been brewing ever since the unholy days of Joseph McCarthy's attempts to censor artistic expression, and the tumultuous, iconoclastic Berkeley "Free Speech Movement" which seemed to rise out of McCarthy's defeat like a people newly liberated and born again. Although the battlelines and alliances have blurred and shifted somewhat over the decades, American society remains so self-contradictory that at times it seems to acquire the aura of cultural schizophrenia. Millions of viewers nightly watch American television although its depiction of human life is overwhelmingly sexual and violent, a portrayal produced and powered by the seemingly omnipotent engines of commerce and capitalism. Other millions of Americans view themselves as the religious faithful, condemning what seems to them a degradation and destruction of the American mind and American morality by a popular culture so graphic and explicit that it reaches a level of obsession they call "satanic." (It is also probably true that many of television's millions of viewers are among the millions attending services every week, and vice versa.) Q1 The author's main point about cultural differences is that: A. differences exist at most levels of human civilization. B. the duality between east and west is the origin of all cultural differences. C. formerly bitter divisions among continents, nations and ethnic groups have relaxed. D. the largest differences are found among competing European societies. Q2 Which of the following influences is/are specifically mentioned as contributing to the reduction of cultural differences? I. Eastern philosophies II. Democratic modernization III. American popular media A. I only B. I and II only C. II and III only D. I, II, and III Q3 European emigration is cited by the author in order to support her suggestion that: A. cultural tensions between European nations eventually became intolerable. B. Marxism-Leninism is not native to Asia. C. conditions in North America led to reconciliation among European cultures. D. American popular culture developed from European traditions. Q4 According to the passage, Joseph McCarthy's attempt to repress cultural expression: A. would be applauded today by Americans who condemn the content of popular culture. B. grew out of the ideological battles of the Cold War. C. successfully promoted cultural conformity in the United States. D. preceded the overthrow of traditional cultural taboos Q5 A contemporary professional architect observed that in the United States, architects "get their qualifications by becoming sterile and say no to all dreams of romance and beauty. They kill their own dreams, so are then allowed to create buildings for mankind." This statement, if true, would most challenge which of following ideas from the passage? A. America has experienced an ongoing cultural contradiction. B. After McCarthy, American culture was able to flower. C. European cultures have become hypnotized by the power of the dollar. D. McDonald's arches are America's best known cultural export. Q6 The passage suggests that the cultural stereotypes held by different western nations about other western nations probably: A. were mostly a means of letting off steam through one-upsmanship. B. reflected deep historical divisions previously addressed through war. C. were reinforced by a tendency to exaggerate and mock differences in others. Correct Answer D. are as widely accepted today as in past centuries.

Q1 A. This is a Main Idea question. A: Yes. See paragraph 1. The author sets out her main point in the opening paragraph, and uses the rest of the passage to elaborate. She notes the cultural "differences between the... western societies, for example, and the...eastern civilizations." She considers the cultural distinctions "among peoples who share a more or less common historical tradition," and finds cultural divisions "even within a single nation." B: No. See paragraphs 4 and 6. The author nowhere suggests that the "cultural differences among western societies," or "within the United States," originate from "the cultural gap between east and west." C: No. While this is true in some cases discussed by the author (see for example the first and fourth paragraphs), the main thrust of the passage is that many differences and divisions remain, including deep divisions within the U.S. (see paragraph 6). D: No. This choice is too narrow to be the main point; the passage also discusses west/east divisions in the first three paragraphs and deep divisions within the U.S. in the last paragraph. Furthermore, this statement is too extreme to be supported by the passage; the author does not indicate that the largest differences are between European nations. Q2 B. This is a Retrieval/New Information question. I: True. See the end of the third paragraph. II: True. See the beginning of the third paragraph. III: False. The popular media, in the form of television, is discussed in the last paragraph. However, it reflects and even contributes to the existence of cultural differences, by broadcasting shows that are offensive to a segment of the population. Q3 C. This is an Inference question. A: No. The author never suggests that the use of cultural stereotypes was primarily ("mostly") a way of releasing anger or frustration ("letting off steam"), as opposed to an expression of people's true beliefs. B: No. See paragraph 4. Although history reveals no lack of brutal warfare among different western nations, nothing in the passage supports this option. C: Yes. The author indicates that "nasty anecdote and sarcastic humor," as well as caricature have long been used to express stereotypes. It is reasonable to infer that the repeated expression of a belief helps to reinforce that belief. D: No. See paragraph 4. If, as the passage states, "most of those tensions among European cultures smoothed over" during more recent generations, then the cultural stereotypes would not likely be so widely accepted today. Q4 C. This is a Structure question. A: No. The author does not suggest that Europeans emigrated to the U.S. because of cultural tensions. She also states that once they settled into the U.S., "most of the tensions... smoothed over." B: No. See paragraph 2. Although the author argues that "in China, for instance, ... Marxism-Leninism... was imposed wholesale from the neighboring behemoth to the north," and that the "philosophy originated in England and Germany," she does not cite European emigration as the means by which the ideology arrived in Asia. This is the right answer to the wrong question. C: Yes. The author suggests that reconciliation occurred among different European cultures "when their emigres came to the western hemisphere and discovered the advantages of living together and learning from each other." D: No. While the author does describe certain U.S. influences on Europe (the lure of the dollar and McDonald's), there is no mention of influence, contemporary or historical, in the other direction. Q5. D. This is a Retrieval question. A: No. Although this option is tempting, the passage makes no specific link between McCarthy's desire for censorship and current condemnations of "immorality" on television. This choice is too strong, and relies on outside knowledge or opinion. B: No. Be careful not to use outside knowledge. Based only on the passage, all we know about McCarthy is that he wanted to "censor artistic expression" and that he was defeated. No connection is made to the Cold War, or even to political ideology. C: No. See paragraph 6. The passage refers to McCarthy's "attempts to censor artistic expression," but strongly implies that those attempts ultimately failed when the "Free Speech Movement" came to life soon after. D: Yes. This answer simply relies on the chronology stated in the passage. Q6. B. This is a New Information question. A: No. The statement in the question stem emphasizes an artistic contradiction in no way contrary to the idea of an ongoing cultural contradiction. B: Yes. If society requires architects to sterilize their imaginations and kill their dreams of romance and beauty before it permits them to create, then the idea that American culture has been able to flower in the post-McCarthy era would be challenged. C: No. The statement in the question stem about American architecture has no direct relevance to Europeans' attraction to the dollar. D: No. McDonald's' Arches could be called architecture; however, this statement made by the author in the passage (paragraph 5) is not inconsistent with the architect's lament in the question stem about the sterility of architecture today. If anything, it supports it.

All things considered, the very problems inherent in the U.S./Mexico relationship may provide possible avenues towards innovative resolution. Since at least the early part of this century the majority of Mexican people have recognized the ugly truth about historical American exploitation of their country. The Mexican-American War, in which Colonel Travis and Davy Crockett made their heroic fighting stand, for American schoolchildren still represents the fundamental values of American patriotism, valor, and freedom: "Remember the Alamo" reverberates through every American heart every time an American president deems it necessary to "draw a line in the sand." Mexican schoolchildren, however, learn an entirely different truth: that the great imperialists to the north invaded sovereign Mexican territory and swallowed up vast parts of the country, including the fabulously wealthy states of gold-laden California and oil-drenched Texas. Over the years, "Uncle Sam" has become such a convenient and credible scapegoat that he was hauled out to the shed any time any Mexican government needed a likely whipping boy to take the popular heat off of some particularly outrageous domestic political betrayal. (Indeed, a political slogan often heard in Mexico, by which certain segments of the opposition have gained considerable popularity in recent times, is, "¡No a la deuda!"—No to the debt!—reflecting the belief strongly held in some quarters that in the currency of exploited natural resources Mexico long ago paid—and overpaid—to the United States whatever revenues it might have subsequently garnered from American banks in the form of loans.) Recent Mexican leaders gave assumed considerable political risk in campaigning for a new relationship with the north. After all those generations of deception by, and distrust for, the americano, to suggest that now the United States could be trusted to become a good neighbor, whose polity would shift from exploitation to partnership, required a good deal of political leadership and personal courage. That fortitude becomes all the more striking when placed in the context of rising dissent among Mexican politicians, and escalating discontent among the people. Mexico does struggle with difficult social problems: massive unemployment, soaring birth rates, a woefully inadequate health care system, and the kind of grinding poverty which demoralizes and dehumanizes existence. On top of all that, and in the face of a severe shortage of hard currency, Mexico still owes massive debts to foreign lenders. Unlike many other developing nations, however, Mexico sits astride enormous reserves of crude oil. And therein, perhaps, lies a way forward. Mexican leaders desperately need debt relief, and to conserve dwindling stores of dollars, they also need to marshal Mexico's limited resources towards rapid industrial development, and at the same time provide some measure of socioeconomic respite to its long-suffering people. An agreement by Mexico, which is not a member of OPEC, to barter its outstanding debts to American financiers for vast quantities of crude oil significantly discounted from the market price, would serve the vital purposes of both nations. By paying with oil, Mexico could save scarce dollars for crucial domestic investment and essential social needs. An influx of cheap petroleum onto the U.S. market would exert considerable downward pressure on domestic U.S. oil prices, resulting in an across-the-board negative multiplier effect on the economy: lowering both producer and consumer prices, significantly reducing industrial costs, creating thousands of new jobs, and injecting a new boost of sustainable consumer confidence and spending into the equation. Moreover, under the newly discounted price structure, a federal tax on gasoline would hardly be noticed, and part of the increased revenues might be used to assist the Mexicans with new environmental protection costs incurred under NAFTA side agreements. Q1 The author of the passage would probably most strongly oppose: A. a move to make Mexico an OPEC member. B. strict enforcement of NAFTA's environmental regulations. C. extending further loans and credits to Mexico. D. development of Mexican industrial capacity. Q2 Which of the following statements, if true, would most undermine the potential effectiveness of the author's proposal outlined in the final paragraph? A. The cheaper imported oil is, the weaker the incentive is to replace fossil fuels with alternatives such as wind and solar power. B. Mexico currently sells its oil to a wide variety of Latin American nations for a higher price than would be paid by the U.S. under the author's proposal. C. The influx of cheap imported oil into the U.S. market would have a stronger effect on producer prices than on consumer prices. D. A decline in industrial costs due to cheaper oil would allow most manufacturing plants to heavily invest in labor-saving machinery and to lay off a large percentage of their current work force Q3 The author's argument that increasing the petroleum supply on the U.S. market will result in an overall strengthening of the economy suggests that the author: I.believes in the mechanics of supply and demand. II is concerned about the environmental dangers of burning fossil fuels. III sees new job creation as important to a healthy economy. A. II only B. I and III only C. II and III only D. I, II, and III Q4 Which of the following factors is/are explicitly cited in the passage to argue for a historic new relationship between Mexico and the United States? I. Past military conflicts II. Past American exploitation of Mexican natural resources III. Mexican resolve to end the long practice of "blame America" rhetoric A. I only B. III only C. II and III only D. I, II, and III Q5 Which of the following findings, if true, best supports the author's assertion that if the policies outlined in the passage were followed, a federal tax on gasoline "would hardly be noticed?" A. Mexican environmental protection efforts depend on U.S. assistance. B. Without the policy change, inflation would raise prices even more than a tax. C. With the new federal tax, cost per gallon of gasoline would be lower than current gas prices. D. Gasoline price increases historically have had little impact on U.S. gas consumption. Q6 The author indicates that the suggestion that Mexico should declare null and void its current outstanding foreign debts is a consequence most directly of which of the following factors? A. Resentment created by the Mexican-American War B. Grinding poverty and hunger in Mexico's urban and rural areas C. Contemporary Mexican political ambition D. Years of corrupt and irresponsible economic administration in Mexico

Q1 A. This is an Inference question. A: Yes. By writing that "Mexico, which is not a member of OPEC" could enter into the described agreement with the U.S., the author suggests that such an agreement might not be possible if Mexico were in fact an OPEC member. Therefore, one can infer that out of the four choices given the author would be most opposed to the idea of Mexico joining OPEC. B: No. See paragraph 6. Nothing in the passage suggests that the author would oppose strict enforcement of NAFTA environmental rules. On the contrary, the author says that "increased revenues might be used to assist the Mexicans with new environmental protection costs incurred under NAFTA." C: No. While loans and credits extended in the past have contributed to Mexico's current debt problems, the author is proposing an entirely new trade relationship with the U.S. There is no evidence in the passage that under this new relationship Mexico would have the same problems in managing its debt. D: No. See paragraph 5. The author recognizes that Mexico needs to marshall "limited resources towards rapid industrial development," and proposes his oil-for-debt plan as a way to achieve that goal. Q2 D. This is a Weaken question. A: No. The author never advocates the replacement of fossil fuels with alternative sources. Therefore, the fact that replacement might become less likely to occur has no impact on the author's proposal. B: No. The author already admits that Mexico would be selling oil to the U.S. at a discounted price. This is an integral part of the author's plan; this choice does not include enough information to show that it is problematic. C: No. The author states that both categories of prices would be lowered. The proposal, as described, does not assume that both would fall by the same amount, nor that consumer prices would fall more. This choice has no effect on the author's argument. D: Yes. If the cost savings described by the author would also stimulate manufacturing industries to replace a large number of workers with machines, it is unlikely that there would be a significant net increase in jobs. Thus, this choice goes furthest towards undermining the potential effectiveness of the proposal. Q3B. This is an Inference/Roman numeral question. I: True. The author argues that the availability of cheap Mexican oil would depress domestic oil prices. II: False. The author never mentions environmental damage specifically from burning oil. Be careful not to use outside knowledge. III: True. One of the author's goals in implementing his proposal is to create "thousands of new jobs" which would then boost consumer confidence and increase spending. Q4 D. This is a Structure question. I: True. See the middle of paragraph 1. II: True. See the the second sentence of paragraph 1. III: True. See paragraph 3. Q5. C. This is a Strengthen question. A: No. There is no connection drawn by the author, nor in this finding, between the protection of the environment in Mexico and how noticeable the new tax might be. B: No. This choice gives no reason to believe the tax would not be noticed. If the proposal were implemented, there would not be inflation but there would be an additional tax. What would happen without the proposal is irrelevant to evaluating the effects of the tax included within the proposal. C: Yes. See paragraph 6. The author claims that "under the newly discounted price structure" a new "tax on gasoline would hardly be noticed," implying that the after-tax price-per-gallon would still be lower than current prices. D: No. The fact that people might continue to buy as much gasoline after the imposition of a tax does not indicate that they would not notice the tax in the first place. Q6.C. This is an Inference question. A: No. While this historical resentment may be a distant contributing factor, it is not described as the most direct cause. B: No. See paragraph 4. The author refers to "the kind of grinding poverty which demoralizes and dehumanizes existence," but does not link that tragic social problem directly with the radical suggestion that Mexico nullify its debts. C: Yes. See paragraph 2. The author states that some factions of the political "opposition... have gained considerable popularity in recent times" by advocating debt nullification. D: No. As in choice A, this history may have contributed to the nature of the Mexican political system, but it is not the most direct cause of calls for voiding the debt.

In the past, when anthropologists thought about protohumans, they usually had in mind the lifeways of beings similar to the technologically primitive hunters and gatherers of today. Early hominids came to be regarded as primitive versions of modern hunter-gatherers: not quite human, but almost. Consequently, whenever anthropologists came across an "uncompromising jumble of bones and stones," it spoke in clear tones of hunter-gatherers. Such "unpromising" piles as the famous sites that Mary Leakey excavated at Olduvai Gorge in Tanzania are found in the record as far back as two million years. Humans, it was inferred, therefore, have been hunters and gatherers for much of their history. "It seemed like common sense," said Isaac. The notion of hominids as hunters has a long intellectual history, going back as far as Darwin's penetrating insights in his 1871 The Descent of Man. "If it be an advantage to a man to have his hands and arms free and to stand firmly on his feet, of which there can be no doubt from his pre-eminent success in the battle for life," he wrote, "then I can see no reason why it should not have been advantageous to the progenitors of man to have become more erect or bipedal. They would thus have been better able to have defended themselves with stones or clubs, or to have attacked their prey or otherwise have obtained their food." For Darwin, hunting was, therefore, a good part of what made us human in the first place, and an admirable part at that. "Darwin's vision of our early hominid ancestors is both familiar and comforting" notes Pat Shipman, an anthropologist at John Hopkins University, Baltimore. "It casts the first human as a noble savage, in tune with his environment, killing only what he needed for him and his family." The hunting theme persisted through many decades and only recently has it been seriously questioned. The result of this questioning has, for some anthropologists at least, been a dramatic shift in perceptions. Our two-million-year-old ancestors weren't hunters at all: They were scavengers. "This second image, of man the scavenger, is both unfamiliar and unflattering," says Shipman. "There is little nobility in man the scavenger." Gone the noble hunter. Gone the sociable food sharer. Gone even the meat eater of any significant degree. Enter the marginal scavenger, the scrounger of leftovers. "They were not romantic ancestors, in the modern sense," admits Binford, "but eclectic feeders commonly scavenging the carcasses of dead ungulates for minor food morsels." This shift occurred in the mid-1970s. At about the same time, other archeologists, notably Lewis Binford of the University of New Mexico, were beginning to address the same question. Binford came to question the interpretations about early hominid lifeways after his scrutiny of a period later in our history, specifically that of the Neandertals. "In the 1960's I had come to the conclusion that the record from Neandertal sites was qualitatively different from the remains left by our own species-modern individuals of both ancient and contemporary forms," he explains. "The more I learned about hunting and characteristic archeological structures for typically modern human ways of life, the more I was convinced that ancient human beings — the Neandertals — had been very different from us. If this was true, then the cozy picture of very early hominids painted by Leaky and Isaac for a much earlier time period appeared to be paradoxical." Whenever a ruling paradigm in science is overturned, the initial phases of its replacement often tend to the opposite extreme. In this case, the noble hunter was replaced by the skulking scavenger. "It's the familiar swing of the pendulum," says Richard Potts. "It goes from one extreme to the other. In fact, if the Olduvai hominids were scavengers in the way Binford describe — and I would have to call that scrounging, not scavenging— they would have been very unusual animals because most scavengers also hunt to some degree." ...No single pattern of behavior can be described as the pattern of hominid behavior. In any case, anthropologists' views on early hominid behavior have undergone a major shift during these recent years; our ancestors now appear to be more like apes than primitive versions of ourselves. Q1. Which of the following best expresses the main idea of the passage? A. There was always a consensus among anthropologists that hominids were hunter-gatherers. B. Neandertals were a step in the evolutionary chain that came after the early hominids. C. The anthropological community discovered direct evidence that early hominids were not hunter-gatherers, but were actually scavengers, taking food from dead carcasses of animals. D. There has been a transformation in the way that anthropologists view early hominid behavior. Q2. The author most likely refers to "lifeways of beings" (paragraph 1) in order to: illustrate a phase in the evolution of modern human beings from early hominids. refer to the manner with which living beings carry out their existence. critique an anthropological hypothesis. A. I only B. II only C. I and II only D. II and III only Q3 Which of the following, if true, would most undermine the author's argument that the shift from the view of protohumans as hunter-gatherers to a belief that early hominids survived through scavenging represented a degradation in the image of these hominids? A. A reliable historical study shows that Darwin believed that early hominids may have practiced some form of agriculture. B. Lions and leopards, which live largely through hunting live prey, are popularly viewed as the noblest of all animals. C. A recent scientific study has proven that early hominids were in fact, hunter-gatherers. D. Vultures and other scavengers have commonly been highly respected as a necessary and valuable factor in maintaining a healthy ecosystem. Q4. The passage indicates which of the following to be true of Neandertals? A. They did not coexist with early hominids. B. They were early hominids. C. They were not hunter-gatherers. D. They were not scavengers Q5. The author indicates that Darwin's view of hominids was: A. of the same persuasion as that of the current anthropological community. B. groundbreaking in that he presented new data on the subject. C. positive because he viewed them as hunters. D. inconsequential since little field research had been done at that point. Q6 The author's representation of the anthropological community's beliefs about hominids is most similar to which of the following? A. The historical community deciding that ancient Native Americans practiced irrigation when they were previously thought not to do so by the majority of historians B. The scientific community's belief concerning the origins of humanity changing from divine creation to Darwin's theory of evolution C. A researcher found a new species of bacteria D. The discovery of gravity

Q1.D Q2.B. This is a Structure/Roman Numeral question. I: False. Included numerals must refer to the function of the cited words in the context of the relevant paragraph as well as of the passage as a whole. While allusions to evolution occur at different points in the passage (for example, paragraph 2 in discussion of Darwin, paragraphs 3 and 6 with references to "ancestors," and paragraph 4 with reference to "ancient human beings"), there is no discussion of evolution in paragraph 1. The author refers to "lifeways of beings" in that first paragraph not to discuss changes in those lifeways or ways of living over time, but rather in order to introduce a discussion of changes in anthropologists' views regarding those lifeways. II: True. "Lifeways of beings" in this context refers to ways in which protohumans existed and survived, be it by hunting and gathering or by scavenging or by some combination of the two. III: False. While critiques of anthropological theories regarding "lifeways of beings" are discussed in the passage, that discussion does not begin until the third paragraph. Any numerals included in the correct answer must represent the function of the cited words in the context of the paragraph in which they appear. Q3.D. This is a Weaken question. Note: The question asks what would most undermine the view that a change in beliefs about early hominids' behavior (hunting-gathering to scavenging) represented a shift from a positive image to a negative one as suggested for example in paragraph 3: "'This second image, of man the scavenger, is both unfamiliar and unflattering,' says Shipman. 'There is little nobility in man the scavenger.' Gone the noble hunter. Gone the sociable food sharer. Gone even the meat eater of any significant degree. Enter the marginal scavenger, the scrounger of leftovers. 'They were not romantic ancestors, in the modern sense,' admits Binford, 'but eclectic feeders commonly scavenging the carcasses of dead ungulates for minor food morsels.'" Therefore, the correct answer has to do the following: 1) relate to this shift from a positive to a negative image, not just to evidence regarding what may have factually been true about early hominids, and 2) go in the right direction; that is, weaken rather than strengthen. A: No. While Darwin's views are cited in the second and third paragraphs as part of the author's argument that hunter-gatherers were seen as noble, this choice does not significantly undermine that representation. There is nothing to suggest that practicing agriculture would be seen as less than noble, or that hunting and agriculture are mutually exclusive. B: No. This choice strengthens rather than weakens the author's argument that hunting was seen as noble. C: No. This choice regards hominids' actual behavior, rather than beliefs held by anthropologists regarding that behavior and the images that went along with those beliefs. If hominids actually were hunter-gatherers, it would have no impact on the author's claims in the passage about anthropologists' beliefs. D: Yes. If scavengers are and have been respected as necessary and valuable, this would undermine the author's claim that when hominids came to be seen a scavengers, this created an "unflattering" or ignoble image. Note that the question requires you to take the information in the answer choice as true; even if you see vultures and other scavengers as less than noble, you can't take that into account when answering the question. While this choice does not prove the author's argument to be false, it is the only one of the four choices that is at all inconsistent with it. Q4.A. This is an Inference question. A: Yes. In paragraph 3, the author writes: "Binford came to question the interpretations about early hominid lifeways after his scrutiny of a period later in our history, specifically that of the Neandertals. 'In the 1960's I had come to the conclusion that the record from Neandertal sites was qualitatively different from the remains left by our own species-modern individuals of both ancient and contemporary forms,' he explains. 'The more I learned about hunting and characteristic archeological structures for typically modern human ways of life, the more I was convinced that ancient human beings-the Neandertals-had been very different from us. If this was true, then the cozy picture of very early hominids painted by Leaky and Isaac for a much earlier time period appeared to be paradoxical.' [emphasis added]" This wording indicates that Neandertals came after early hominids, and that they did not coexist. B: No. In paragraph 4, early hominids are clearly distinguished from Neandertals (see passage cite for solution for choice A). C: No. While it is suggested that Binford (paragraph 4) believed Neandertals may not have been hunter-gatherers, the author does not express agreement with this belief. Note that the author never takes sides with one or the other of the "extreme" views presented in the passage, and in the last two paragraphs the author indicates that it is unlikely that the hominids were either only scavengers or only hunter-gatherers. The quote from Potts in paragraph 5 suggests that even scavengers may also hunt, and the author himself states in the last paragraph that "No single pattern of behavior can be described as the pattern of hominid behavior." D: No. The author gives evidence in the fifth paragraph that it is possible that the early hominids both hunted and scavenged, and in the last paragraph the author writes: "No single pattern of behavior can be described as the pattern of hominid behavior." Therefore, both choices C and D have the same problem; the passage supports neither "extreme" (that is, only one or the other behavior) view. Q5.C. This is an Inference question. A: No. Darwin believed that hominids were hunters (paragraph 2), while a major theme of the passage is that at some point the anthropological community came to believe (starting in the 1970's) that early hominids were not in fact hunters. Darwin is given in the passage as a point of contrast with anthropologists' later and current beliefs. B: No. There is no new data cited or described. Paragraph 2 simply explains Darwin's thoughts, based on advantages of being bipedal. C: Yes. The second paragraph indicates that Darwin believed hominids or "the progenitors of man" to have been hunters, and at the end of the paragraph he writes: "For Darwin, hunting was therefore a good part of what made us human in the first place, and an admirable part at that." In the third paragraph, the author writes: "'Darwin's vision of our early hominid ancestors is both familiar and comforting' notes Pat Shipman, an anthropologist at John Hopkins University, Baltimore. 'It casts the first human as a noble savage, in tune with his environment, killing only what he needed for him and his family.'" D: No. The author makes no negative judgment about the importance of Darwin's work, and the passage does not indicate how much field work might have been done in or by Darwin's time. Q6.A. This is an Analogy question. A: Yes. This choice represents a very similar shift in beliefs about how a certain people "made their living" or survived. B: No. While this choice represents a shift in belief within a community, it is on a very different scale than the change described in both the passage and choice A. While the change is described in the passage as causing a "dramatic shift in perceptions", that change regards only the nature of how hominids survived in a certain period of human history. This answer choice describes a shift in beliefs regarding the very origins of human life. If you compare choice A to choice B, choice A is a much closer fit to the passage. C: No. There is no discussion in the passage of discovering something entirely new (such as a new species of proto-humans). The shift in the passage only regards beliefs about something we already knew existed. D: No. As in choice C, there is nothing that is entirely new being discovered in the passage. Rather, the author discusses changes in beliefs about something that has already been "discovered."

Human knowledge has two forms: it is either intuitive knowledge or logical knowledge; knowledge obtained through the imagination or knowledge obtained through the intellect; knowledge of the individual or knowledge of the universal; of individual things or of the relations between them: it is, in fact, productive either of images or of concepts. In ordinary life, constant appeal is made to intuitive knowledge. It is said to be impossible to give expression to certain truths; that they are not demonstrable by syllogisms; that they must be learnt intuitively. The politician finds fault with the abstract reasoner, who is without a lively knowledge of actual conditions; the pedagogue insists upon the necessity of developing the intuitive faculty in the pupil before everything else; the critic in judging a work of art makes it a point of honor to set aside theory and abstractions, and to judge it by direct intuition; the practical man professes to live rather by intuition than by reason. But this ample acknowledgment, granted to intuitive knowledge in ordinary life, does not meet with an equal and adequate acknowledgment in the field of theory and of philosophy. There exists a very ancient science of intellective knowledge, admitted by all without discussion, namely, Logic; but a science of intuitive knowledge is timidly and with difficulty admitted by but a few. Logical knowledge has appropriated the lion's share; and if she does not quite slay and devour her companion, yet yields to her with difficulty the humble little place of maidservant or doorkeeper. What, it says, is intuitive knowledge without the light of intellective knowledge? It is a servant without a master; and though a master finds a servant useful, the master is a necessity to the servant, since he enables him to gain his livelihood. Intuition is blind; Intellect lends her eyes. Now, the first point to be firmly fixed in the mind is that intuitive knowledge has no need of a master, nor to lean upon any one; she does not need to borrow the eyes of others, for she has most excellent eyes of her own. Doubtless it is possible to find concepts mingled with intuitions. But in many other intuitions there is no trace of such a mixture, which proves that it is not necessary. The impression of a moonlight scene by a painter; the outline of a country drawn by a cartographer; a musical motive, tender or energetic; the words of a sighing lyric, or those with which we ask, command and lament in ordinary life, may well all be intuitive facts without a shadow of intellective relation. But, think what one may of these instances, and admitting further that one may maintain that the greater part of the intuitions of civilized man are impregnated with concepts, there yet remains to be observed something more important and more conclusive. Those concepts which are found mingled and fused with the intuitions, are no longer concepts, in so far as they are really mingled and fused, for they have lost all independence and autonomy. They have been concepts, but they have now become simple elements of intuition. The philosophical maxims placed in the mouth of a personage of tragedy or of comedy, perform there the function, not of concepts, but of characteristics of such personage; in the same way as the red in a painted figure does not there represent the red color of the physicists, but is a characteristic element of the portrait. The whole it is that determines the quality of the parts. A work of art may be full of philosophical concepts; it may contain them in greater abundance and they may be there even more profound than in a philosophical dissertation, which in its turn may be rich to overflowing with descriptions and intuitions. But, notwithstanding all these concepts it may contain, the result of the work of art is an intuition; and notwithstanding all those intuitions, the result of the philosophical dissertation is a concept. The Promessi Sposi contains copious ethical observations and distinctions, but it does not for that reason lose in its total effect its character of simple story, of intuition. In like manner the anecdotes and satirical effusions which may be found in the works of a philosopher like Schopenhauer, do not remove from those works their character of intellective treatises. The difference between a scientific work and a work of art, that is, between an intellective fact and an intuitive fact lies in the result, in the diverse effect aimed at by their respective authors. This it is that determines and rules over the several parts of each. Q1. Which of the following best represents the main idea of the passage? A. The two main forms of human knowledge, intuition and logic, appeal to different types of people: intuition is the province of the artists, critics and teachers, while intellect is the master of politicians, scientists and philosophers. B. Although intuitive knowledge is aptly used in ordinary life, it is inadequately acknowledged in the field of theory, despite the fact that it is just as valid, if not more so, than intellectual knowledge, in both arenas. C. Intuition and intellect exist in all forms of human thought, both creative and scientific - while both are important oppositional forces, intellect is the master of intuition. D. While the two forms of human knowledge are not equally acknowledged in every field, neither is necessarily superior - in fact, the two are often mingled and fused such that it is most appropriate to discern the difference between intellect and intuition by deferring to the intent of the source. Q2. All of the following are used by the author to describe intuitive knowledge EXCEPT: A. individual things. B. abstraction. C. images. D. maidservant. Q3. Suppose that a theoretical engineer wrote a paper on the philosophy of science that, in addition to numerous abstractions, also contained many imaginative descriptions. How would the author of this passage categorize this work? A. As the work of a practical man, attempting to marry intuition and reason. B. As a piece of intuitive art that seamlessly combines logic with intuition. Your Answer C. As an intellective concept. D. This author would probably not have a strong opinion about the piece either way, considering that the author is merely concerned with philosophical reasoning and not the specific sciences. Q4. Which of the following pairs is NOT represented as opposites in this passage? A. Ordinary life: philosophy B. The Promessi Sposi: Schopenhauer C. Cartographer: painter D. Intuition: logic Q5. Which of the following, if true, would most undermine the author's claim in the last paragraph about the difference between scientific work and artistic work? A. Today, most art, especially that produced before the turn of the century, is considered artifact, and is therefore carefully preserved and studied to learn more about the cultures and peoples that produced these pieces and to draw universal lessons about human society. B. Many scientists attempt to avoid usage of flowery, descriptive language and reference to personal biases when writing for scientific peer-reviewed journals. C. The most popular trend in modern bestsellers, due to a more sophisticated and educated reading public, has been the incorporation of more scientific ideas—psychology, criminology, sociology and philosophy—into mainstream novels. D. Several recent studies suggest that physicians and other medical professionals most commonly change careers to creative or artistic fields because of job dissatisfaction and burn out. Q6.The author probably uses the color red in the last paragraph in order to: A. further his stance on how colors can represent philosophical concepts in context. B. argue that perception of color depends on the observer; a physicist will perceive color as a scientist might and a painter will perceive color as an artist. C. provide a comparable yet contrasting example to further his argument that the characteristic of the individual pieces is determined by the purpose of the whole. D. explain how one work of art might represent multiple different concepts with colors. Q7.Based only on its use in the passage, what is the most appropriate definition for the term "concept"? A. A logical notion or idea B. An image C. An intuitive syllogism D. A concrete reason

Q1.D. This is a Main Point question. A: No. This choice directly contradicts the passage, and to the extent that it is accurate, is too narrow (only encompassing the first two paragraphs). Politicians, in the second paragraph, are described as relying on intuitive knowledge: "In ordinary life, constant appeal is made to intuitive knowledge. ...The politician finds fault with the abstract reasoner, who is without a lively knowledge of actual conditions." B: No. While the first portion of this answer is supported by the passage, the claim that intuitive knowledge is "just as valid, if not more so, than intellectual knowledge, in both arenas," is not. Additionally, this choice leaves out an important theme: the author's intent in the whole work determines the nature of the parts (last paragraph). C: No. This choice directly contradicts the author's stance in the fourth paragraph: "Now, the first point to be firmly fixed in the mind is that intuitive knowledge has no need of a master, nor to lean upon any one; she does not need to borrow the eyes of others, for she has most excellent eyes of her own." D: Yes. This answer choice most appropriately summarizes the first few paragraphs, and references the author's final claim in the last paragraph that "The difference between a scientific work and a work of art, that is, between an intellective fact and an intuitive fact lies in the result, in the diverse effect aimed at by their respective authors. This it is that determines and rules over the several parts of each." Q2.B. This is a Structure/Except question. A: No. This choice is supported by the first paragraph: "Human knowledge has two forms: it is either intuitive knowledge or logical knowledge; knowledge obtained through the imagination or knowledge obtained through the intellect; knowledge of the individual or knowledge of the universal; of individual things or of the relations between them." Note the parallel structure of the sentence, which tells you that the first member of each pair is intuitive knowledge, while the second is logical knowledge. B: Yes. Logical knowledge is considered abstract, not intuitive knowledge: "In ordinary life, constant appeal is made to intuitive knowledge.... The politician finds fault with the abstract reasoner, who is without a lively knowledge of actual conditions; ...the critic in judging a work of art makes it a point of honor to set aside theory and abstractions, and to judge it by direct intuition" (paragraph 2). Here "abstraction" is clearly associated with the opposite of intuitive knowledge; therefore this is the best answer for this EXCEPT question. C: No. This choice is supported by the first paragraph: "Human knowledge has two forms: it is either intuitive knowledge or logical knowledge; ... it is, in fact, productive either of images or of concepts." D: No. This choice is supported by the third paragraph: "Logical knowledge has appropriated the lion's share; and if she does not quite slay and devour [intuitive knowledge], yet yields to her with difficulty the humble little place of maidservant." Q3.C. This is a New Information question. A: No. There is no evidence in the passage that the author might classify the engineer as a "practical man." On the contrary, the passage does provide evidence as to how the author might categorize his work: "and notwithstanding all those intuitions, the result of the philosophical dissertation is a concept" (paragraph 5). B: No. Quite the opposite, the author would not consider this a piece of art at all, but rather a piece of logic: "The whole it is that determines the quality of the parts. A work of art may be full of philosophical concepts; it may contain them in greater abundance and they may be there even more profound than in a philosophical dissertation, which in its turn may be rich to overflowing with descriptions and intuitions. But, notwithstanding all these concepts it may contain, the result of the work of art is an intuition; and notwithstanding all those intuitions, the result of the philosophical dissertation is a concept" (paragraph 5). C: Yes. The portion of the passage that most directly explains this new information is the final paragraph, which states: "The whole it is that determines the quality of the parts. A work of art may be full of philosophical concepts; it may contain them in greater abundance and they may be there even more profound than in a philosophical dissertation, which in its turn may be rich to overflowing with descriptions and intuitions. But, notwithstanding all these concepts it may contain, the result of the work of art is an intuition; and notwithstanding all those intuitions, the result of the philosophical dissertation is a concept." Therefore the author would most likely categorize the work of this theoretical engineer as a concept or a product of intellect. D: No. This passage definitely provides enough examples to demonstrate that the author has considered many different types of arts and sciences and would probably have an opinion about the information provided in this question stem. Based on the final paragraph, the author would most likely categorize the engineer's paper as a concept: "and notwithstanding all those intuitions, the result of the philosophical dissertation is a concept" (paragraph 5). Q4.C. This is an Inference/Not question. A: No. The author sharply contrasts ordinary life and philosophy by stating that "in ordinary life, a constant appeal is made to intuitive knowledge" (paragraph 2) and "But this ample acknowledgment, granted to intuitive knowledge in ordinary life, does not meet with an equal and adequate acknowledgment in the field of theory and of philosophy" (paragraph 3). This implies that there is a sharp contrast between the two. B: No. The author uses these as two contrasting examples in paragraph 5: "The Promessi Sposi contains copious ethical observations and distinctions, but it does not for that reason lose in its total effect its character of simple story, of intuition. In like manner the anecdotes and satirical effusions which may be found in the works of a philosopher like Schopenhauer, do not remove from those works their character of intellective treatises." C: Yes. This is the best answer because it is the only one that is not contrasted in the passage - in fact, these two are used in the same sentence as comparable examples: "The impression of a moonlight scene by a painter; the outline of a country drawn by a cartographer... may well all be intuitive facts without a shadow of intellective relation" (paragraph 4). D: No. These two ideas are contrasted throughout the passage, for example: "Human knowledge has two forms: it is either intuitive knowledge or logical knowledge" (paragraph 1). Q5.A. This is a Weaken question. A: Yes. The author's claim is that "The difference between a scientific work and a work of art, that is, between an intellective fact and an intuitive fact lies in the result, in the diverse effect aimed at by their respective authors" (paragraph 5), implying that it is the INTENT of the producer of the work that should determine if it is a scientific work or a work of art. This answer choice significantly undermines this contention by demonstrating that the audience determines the effect of the piece (not the "respective authors") by using the artwork as a scientific tool to learn more about the people and cultures that produced it. Therefore, this choice contradicts the author's contention by demonstrating that a work of art could be utilized as a scientific work to gain universal (rather than specific) knowledge, despite the intent of the creator of the artwork. B: No. This choice implies that scientists are seeking to eliminate any traces of "intuitive" elements from their scientific work, thereby neither supporting nor undermining the author's stance in the final paragraph. C: No. This choice is tempting because it references a combination of artistic and scientific elements. However, the author's main stance is that, despite what logical or scientific elements may be in a piece of art, the overall product is determined by the intent of the author; therefore, it is still a work of art: "The Promessi Sposi contains copious ethical observations and distinctions, but it does not for that reason lose in its total effect its character of simple story, of intuition" (paragraph 5). Thus, there is nothing in this choice that is inconsistent with the author's argument. D: No. This choice is not directly relevant to the author's argument that the difference between a scientific work and work of art lies in the intent of the respective authors. Q6.C. This is a Structure question. A: No. The author never claims that colors can "represent philosophical concepts." This answer is taking many different ideas out of context and making a comparison that the author does not make in this text. B: No. The author does not conclude that the "perception of color depends on the observer." This answer takes this example out of context. The issue in the passage is the intent of the artist, not the perception of the observer. C: Yes. The author states that: "The philosophical maxims placed in the mouth of a personage of tragedy or of comedy, perform there the function, not of concepts, but of characteristics of such personage; in the same way as the red in a painted figure does not there represent the red color of the physicists, but is a characteristic element of the portrait." (paragraph 5). Therefore this example of the color red is comparable to, yet contrasting with, the example of the "personage of tragedy or of comedy," and illustrates the author's conclusion that "the whole it is that determines the quality of the parts." D: No. While part of this idea is presented later in the paragraph ("A work of art may be full of philosophical concepts" (paragraph 5)), this is not connected specifically to the use of color. Therefore, this choice does not give an accurate description of why the author mentions the color red. Q7.A. This is an Inference question. A: Yes. The term "concept" is used in many places, but is clearly associated with logic in the first paragraph: "Human knowledge has two forms: it is either intuitive knowledge or logical knowledge... it is, in fact, productive either of images or of concepts." Therefore this choice is directly supported by the passage, and makes the most sense in the context of where and how "concept" is used throughout the passage. B: No. Concepts and images are distinguished from each other in the first sentence of the passage, which provides a series of related contrasts:: "Human knowledge has two forms: it is either intuitive knowledge or logical knowledge... it is, in fact, productive either of images or of concepts." C: No. Even without necessarily knowing what the word "syllogism" means (a formal deductive argument), it is possible to eliminate this as a potential answer because the choice itself is contradictory. The passage states that: "It is said to be impossible to give expression to certain truths; that they are not demonstrable by syllogisms; that they must be learnt intuitively" (paragraph 2), which implies that a syllogism and intuition are two opposite concepts. D: No. The term "concept" is associated with logical knowledge: "Human knowledge has two forms: it is either intuitive knowledge or logical knowledge... it is, in fact, productive either of images or of concepts" (paragraph 1), and logical knowledge is described as abstract: "the critic in judging a work of art makes it a point of honor to set aside theory and abstractions, and to judge it by direct intuition" (paragraph 2). Therefore, because "concrete" is the opposite of "abstract," this choice is definitely not the best synonym forthe term "concept."

Snow White is carried off by the prince, inert in her coffin; it is by chance that she coughs up the poisonous piece of apple stuck in her throat and thus comes back to life. Sleeping Beauty awakens only because her lover kisses her. Cinderella's time of degradation ends when the slipper fits her. In each of these stories—as in so many others—the rescuer demonstrates his love for his future bride in some form. We are left in the dark about the feelings of the heroines, however. The way the Brothers Grimm tell these stories, we hear nothing about Cinderella being in love, although we may draw some conclusions from the fact that she goes to the ball three times to meet her prince. About Sleeping Beauty's feelings we learn only that she looks "in a friendly fashion" at the man who frees her from her enchantment. Similarly, all we are told is that Snow White "felt friendly" toward the man who brought her back to life. It seems as if these stories deliberately avoid stating that the heroines are in love; one gets the impression that even fairy tales put little stock in love at first sight. Instead, they suggest that much more is involved in loving than being awakened or chosen by some prince. The rescuers fall in love with these heroines because of their beauty, which symbolizes their perfection. Being in love, the rescuers have to become active and prove that they are worthy of the woman they love—something quite different from the heroine's passive acceptance of being loved. In "Snow White" the prince declares he cannot live without Snow White, he offers the dwarfs whatever they want for her, and is finally permitted to carry her off. In penetrating the wall of thorns to reach Sleeping Beauty, her suitor risks his life. The prince in "Cinderella" devises an ingenious scheme to trap her, and when he catches not her but only her slipper, he searches for her far and wide. The stories seem to imply that falling in love is something that happens; being in love demands much more. But since the male rescuers in these stories have only supporting roles, nothing more specific can be learned from their behavior about what developments are involved in loving somebody, what the nature of the commitment "being in love" entails. All the stories considered so far convey that if one wishes to gain selfhood, achieve integrity, and secure one's identity, difficult developments must be undergone: hardships suffered, dangers met, victories won. Only in this way can one become master of one's fate and win one's kingdom. What happens to the heroes and heroines in fairy tales can be likened—and has been compared—to initiation rites in which the novice enters naïve and unformed, and which dismiss him at their end on a higher level of existence undreamed of at the start of this sacred voyage through which he gains his reward or salvation. Having truly become himself, the hero or heroine has become worthy of being loved. But meritorious as such self-development is, and while it may save our soul, it is still not enough for happiness. For this, one must go beyond one's isolation and form a bond with the other. On however high a plane his life may proceed, the I without the Thou lives a lonely existence. The happy endings to fairy tales, in which the hero is united with his life's partner, tell this much. But they do not teach what the individual must do to transcend his isolation after he has won his selfhood. Neither in "Snow White" nor in "Cinderella" (the Brothers Grimm's versions) are we told anything about their life after they are married; nothing is said about them living happily with their partner. These stories, while they take the heroine up to the threshold of true love, do not tell what personal growth is required for union with the beloved other. Q1 Which of the following statements best summarizes the author's primary purpose in the passage? A. To question the sexism inherent in fairy tales which portray women as passive recipients of men's love B. To warn parents about dangerous omissions of certain life lessons from fairy tales enjoyed by children C. To explore the representation of love and selfhood in some fairy tales D. To argue that "Cinderella," "Snow White," and "Sleeping Beauty" are the most beloved of fairy tales by the Brothers Grimm Q2.Which of the following plot details of the discussed fairy tales is NOT given in the passage? A. Snow White's feelings of friendliness towards the man who brings her back to life B. The prince's fitting a glass slipper on the foot of Cinderella C. Sleeping Beauty's prince risking his life to reach her D. Snow White's coughing up of a piece of poisonous apple Q3. Which of the following words is most synonymous with "meritorious" as it is used in paragraph 4? A. insufficient B. adequate C. difficult D. beneficial Q4 Which of the following fairy tales best fits the author's explanation of love in fairy tales, as presented in the passage? A. "The Little Mermaid," in which a mermaid gives up her voice, tail and immortality for the opportunity to walk on land and gain a prince's love. After enduring many hardships, the mermaid fails in her quest as the prince marries another woman, and the mermaid must die. B. "The Princess and the Pea," in which a girl must sleep on a pile of twenty mattresses and twenty featherbeds on top of a single pea in order to prove her authenticity as a princess; when she complains she was unable to sleep because the pea disturbed her, the prince marries her, knowing only a true princess could be so sensitive. C. "Little Red Riding Hood," in which a young girl trusts a wolf with her grandmother's address. The wolf then eats the grandmother and poses as her in order to eat Little Red Riding Hood. Afterwards, a huntsman comes and slays the wolf, allowing Little Red Riding Hood and her grandmother to escape from its belly unharmed. D. "Beauty and the Beast," in which Beauty offers herself to be Beast's captive in exchange for her father's freedom, even though the beast's appearance leaves her trembling with fear. When Beast allows her to leave his castle and visit her family for a week, the Beast starves himself in anguish over losing her. When she returns to see him almost dead, Beauty's declaration that she loves him transforms the Beast into a handsome prince and the two marry. Q5 According to the passage, which of the following is a necessary condition of the achievement of selfhood in the fairy tales discussed? A. Marrying one's true love B. Recognizing full happiness C. Undergoing difficult developments D. Naivety at the outset of a quest

Q1 C. This is a Primary Purpose question. A: No. While the author calls the female characters in these fairy tales passive recipients of male characters' love (paragraph 2), he does not make any comment on whether this or any other aspect of fairy tales is sexist. B: No. The author mentions that fairy tales often end before giving their readers answers about what happens after marriage, but there is nothing in his tone to suggest his words constitute a warning, nor is there evidence to suggest his intended audience is comprised of parents. C: Yes. The author's tone is one of exploration more than questioning, arguing, or warning, and his main idea involves what these stories suggest about the attainment of selfhood and the nature of love. The first two paragraphs discuss what these stories suggest about the difference between falling and being in love, and the last two paragraphs connect the idea of being in love to selfhood. D: No. The author merely mentions these stories as the basis of his discussion about love and selfhood, but does not comment on the popularity of these fairy tales compared with that of others. Q2 B. This is a Retrieval/NOT question. A: No. This information is provided near the end of the first paragraph. B: Yes. This answer choice adds two details not provided: that the prince is the one who fits the slipper on Cinderella's foot, and that the slipper is made of glass. Beware of using outside knowledge to fill in information that is not actually in the passage. C: No. This information can be found in the middle of the second paragraph. D: No. This information is provided in the passage's first sentence. Q3 D. This is an Inference question. A: No. While the author suggests that "such self development ... is still not enough for happiness," this is a contrasting point with the use of the term meritorious, as indicated by the word "but" preceding it. It would be redundant to say "But insufficient as such self development might be, it is still not enough for happiness." B: No. For the author to suggest that "such self development ... is still not enough for happiness," he cannot also be saying that this self-development is adequate. Substituting "adequate" in the place of meritorious reveals this answer choice as incorrect, since it would say that self development is both adequate and not adequate. C: No. While the author discusses that self development is achieved through difficulty, this is the point of the paragraph preceding the one that includes the word meritorious. In the sentence including the word, the author is clearly setting up a juxtaposition, suggesting that as good as self-development is, it is not quite enough for the attainment of happiness. D: Yes. Using our own answer to the question by noting the contrast inherent in the use of the word "but" at the beginning of the sentence, we know the word meritorious must mean something other than "not enough for happiness." The "enough" indicates that meritorious must be something positive since it does not run contrary to happiness but rather it is not the sole factor necessary in achieving happiness. Q4. D. This is an Analogy question. A: No. The Little Mermaid might endure hardships, but she does not get married at the end of her fairy tale, or "form a bond" that lasts. Therefore, this does not match the author's description in the passage. B: No. This is an attractive answer, since it contains the element of marriage at the end of the story. However, in contrast with Beauty's situation described in choice D, sleeping on a pea covered by many mattresses does not sufficiently qualify as a "difficult development" (paragraph three), nor is it an instance of becoming "active" to "prove that [one is] worthy" of the love of another (paragraph 2). The princess' worthiness of being loved resides only in the proof that she is a princess and not in any difficult development she must undergo. C: No. This story has nothing to do with romantic love or marriage, and therefore has no direct relevance to the passage. D: Yes. In paragraph three, the author states that in order to achieve the selfhood necessary to become worthy of love, "difficult developments must be undergone: hardships suffered, dangers met, victories won." Beauty's offer to give herself to the beast despite her fears qualifies as such a hardship; her demonstration of virtue in her declaration of love parallels the active acts of rescue described by the author in paragraph two. Q5. C. This is a Retrieval question. A: No. Marriage follows the achievement of selfhood according to paragraphs 3 and 4; it is not a condition that precedes it. B: No. Achieving selfhood is one step along the way to achieving full happiness; full happiness is not a condition to selfhood's achievement. See paragraphs three and four. C: Yes. In paragraph three, the author states, "All the stories considered so far convey that if one wishes to gain selfhood, achieve integrity, and secure one's identity, difficult developments must be undergone: hardships suffered, dangers met, victories won. Only in this way can one become master of one's fate and win one's kingdom." D: No. Although in paragraph three the author mentions that what happens to heroes and heroines in fairy tales can be and has been compared to initiation rituals in which one begins in a naïve state, he does not suggest that naivety is a necessary condition to the achievement of selfhood.

Environmental policy choices are made especially difficult by their economic implications. Controlling pollution is a significant economic cost for many firms today. In the essentially nonregulatory era prior to the 1960s, this expense was either quite low or nonexistent. Increasing government regulatory intervention in industrial processes and output reflects the indisputable fact that market forces by themselves will not guarantee the protection of health and environment. Polluters have little or no economic incentive to stop polluting, but clean air and water are public goods that should be left available to all of us. Left unregulated, most firms tend to maximize profits by minimizing costs—including the costs of environmental protection. Determining exactly what constitutes pollution or environmental degradation and deciding how stringently to regulate polluting activities have tremendous economic implications for firms and governments. Too much regulation could depress the economic growth at national, state, and local levels and even force some companies into bankruptcy. The tradeoffs between economic growth and environmental protection spawn conflict, and are evident wherever these important objectives clash. In Oregon, powerful timber interests have to harvest trees in order to preserve the 68,000 jobs they contribute to that state's economy, yet environmentalists won federal court prohibitions on logging where the endangered spotted owl is threatened (loggers responded with T-shirt logos such as "Save a Logger—Eat an Owl.") In Maryland, a 1989 law explicitly recognized and indeed mandated a tradeoff between economics and environment: it requires developers of nontidal wetlands to replace lost swamps and bogs so that "no net loss" of the ecologically valuable wetlands occurs. All of us are potential victims of environmental problems. Polluted air, water, and land offend us aesthetically, but, more important, they threaten the health and safety of ourselves, our children, and our grandchildren. People who drink contaminated water and breathe polluted air experience the costs very directly. From a different perspective, all citizens must help pay the price of a safe and clean environment. We do this through the portion of our taxes that goes to government pollution control efforts and through the prices we pay for goods, which include the cost of pollution control. The costs to consumers are especially heavy for the products of chemical companies, the auto industry, and coal-burning power plants. Not surprisingly, these industries have lobbied heavily against what they perceive to be excessive regulation. Government's role is to balance economic growth with environmental protection by regulating polluters. The political economy of environmental protection argues strongly for national domination of policymaking. Because states and local governments compete for industry, a nonnational jurisdiction might be tempted to relax environmental protection standards in order to influence a firm's decision on where to construct or expand a new manufacturing facility. Only national policies and standards can prevent the sacrifice of environmental quality in jurisdictions that seek growth and development at virtually any cost. By imposing minimum environmental protection standards on all states and localities, the national government helps limit interstate competition for polluting firms, since all prospective polluters must adhere to national standards. According to the passage, the role played by national governments in environmental protection should be to: A. balance economic growth with the needs of industrial companies. B. oversee the state and local governments in regulating their pollution prevention efforts. C. increase the regulation of pollution by industrial companies. D. mediate between environmental and economic decisions. Which of the following examples do the authors NOT use in the passage to illustrate the tradeoff between environmental protection and economic growth? A. Tree harvesting to save jobs B. Heavy producer costs for chemical company products C. Replacement of lost swamps and bogs D. Logging prevention in spotted owl territories What is the authors' main point? A. There should be an increase in the amount of government mediation between economic growth and environmental protection. B. Environmental policy choices are difficult and it is important that the government act as an arbitrator of these decisions on a national level. C. There are many examples of tradeoffs between economic growth and environmental protection that the government should take into account. D. All government should take an interest in drafting and enforcing environmental policy. The passage implies that prior to the 1960s: A. controlling pollution did not have much of an effect on the profits of companies. B. it was very costly for companies to regulate their pollution. C. pollution control was believed to be unnecessary. D. the government did not enforce pollution control because market forces guaranteed the protection of the environment. Which of the following governmental regulatory policies would be most consistent with the authors' stance in the passage? A. A government policy that minimizes its intervention in company affairs and lets the market protect the environment B. A government policy that includes a large amount of strict environmental regulation at all levels C. A policy in which the federal government balances between the economy and the environment D. A government policy that allocates responsibility for environmental policy regulation between the national, state and local levels The passage implies that: A. because too much regulation could cause the economy to decline, there should be less government pollution regulation. B. chemical companies, the auto industry, and coal-burning power plants should have higher levels of pollution regulation than they do now because these companies produce more waste. C. since the 1960s, companies have been paying less for waste management. D. without regulation, a polluter will continue to pollute. Suppose a state's government was concerned with water pollution in their local lakes caused by businesses situated along the coastlines. The state government has contacted the authors in order to help devise a policy to deal with the problem. Which of the following policies would the authors most likely suggest? A. A policy in which the state government strikes a balance between how much water pollution is environmentally safe and economically feasible B. A policy that does not deal with the water pollution issue on the state level, but which appeals to the national government to take control of this environmental problem C. A policy in which taxes are imposed on the polluters to pay for pollution clean-up D. A policy in which the state government does not deal with the issue

Q1 D. This is a Retrieval question. A: No. This is a half right/half wrong answer choice, with the second part being incorrect. According to the second paragraph, most companies want or need to maximize profit. To the extent that profit is relevant, it is a factor that contributes to economic growth, and so these are both on the same side of the equation. The authors state in paragraph 5 and also indicate elsewhere in the passage that "Government's role is to balance economic growth with environmental protection." This answer leaves out the other side of this relationship. B: No. The authors do say that regulation should occur at the national level, but the passage does not specifically state that the national government should regulate lower levels of government. C: No. The authors do not provide an opinion on whether or not regulation should be increased or decreased. D: Yes. The authors write in paragraph 5 that the "government's role is to balance economic growth with environmental protection by regulating polluters." This would involve weighing against each other decisions based on increasing growth and those made to limit pollution. Q2 B. This is an Structure/NOT question. A: No. Paragraph 3 brings up the timber industry and its role in preserving jobs. Since this example is found in the passage and the question asks for what example is not given in the passage, this is incorrect. B: Yes. The authors do NOT use this as an example to illustrate the tradeoff. The authors discuss costs for products from chemical companies, the auto industry, and coal-burning power plants in paragraph 4. However, this is in reference to consumer costs, not producer costs. While the authors do suggest in paragraph 2 that pollution control involves costs to producers, chemical companies are not specifically given as an example. C: No. Paragraph 3 also discusses the law that "requires developers of nontidal wetlands to replace lost swamps and bogs" as an example of this tradeoff. The question asks for which example is not used, therefore this answer choice is incorrect. D: No. Protection of spotted owl territories by prohibiting logging in those areas is discussed as another example in paragraph 3. This answer choice is incorrect because the question asks for which example is not found in the passage. Q3 B. This is a Main Idea question. A: No. While the passage does discuss a balance between economic growth and environmental protection, the authors do not express any belief that this government mediation should be increased. B: Yes. This answer best captures the content, scope, and tone of the passage as a whole. It mentions the authors' belief in what role the government should play in environmental policy while staying neutral on whether or not more or less government involvement is needed. C: No. This answer choice is too narrow. Yes, the passage describes several examples of this tradeoff, but explaining these examples is not the overall purpose of the article. Notice, for example, that this choice leaves out a major issue: the need for national level regulation. D: No. This answer choice is too broad to be the main idea. The authors state that national government should be involved in environmental policy-making, not all government levels. This answer choice also does not address the balance between the economy and the environment. Q4 A. This is an Inference question. A: Yes. In paragraph 1 the authors write that, "prior to the 1960s, this [pollution control] expense was either quite low or nonexistent." If the cost for pollution control was not high, it would not greatly affect the companies' profits. B: No. This is a reversal of what the authors explain: it was actually not very costly for companies to regulate their pollution. C: No. This choice is too extreme. The authors say that there was little regulation before the 1960s, but there isn't any evidence in the passage that this was because it was believed that regulation was unnecessary. Note that while the authors do argue that "market forces by themselves will not guarantee the protection of health and environment," the authors do not suggest that others believed that it does, or that this belief was a reason for the lack of regulation. D: No. Paragraph 1 states that, "market forces by themselves will not guarantee the protection of health and environment." Therefore, this answer choice is a reversal of what the authors are saying. Q5 C. This is an Inference question. A: No. The authors state in paragraph 1 that, "market forces by themselves will not guarantee the protection of health and environment." Therefore, a government that attempted to let the market protect the environment would contradict what the passage states. B: No. The authors argue that regulation should occur at the national level and specifically state in paragraph 5 that, "a nonnational jurisdiction might be tempted to relax environmental protection standards." A policy that included regulation at the local and state levels is not one that the authors would agree with. Also, the authors state that we must balance regulation against economic growth and that too much regulation would be damaging (paragraph 2). Therefore, "a large amount of strict regulation" is too extreme. C: Yes. Paragraph 5 discusses the authors' belief that the role of the government is, "to balance economic growth with environmental protection." Therefore, a policy that calls upon the federal government to act as such a mediator would be in agreement with the passage. D: No. Similarly to answer choice B, this answer discusses a policy in which environmental protection regulation occurs at all levels of government. The authors do not support this type of regulation and therefore would not agree with such a policy. Q6 D. This is an Inference question. A: No. This is a half right/half wrong answer; the first half is correct because the authors state in paragraph 2 that too much regulation of company pollution could affect the economy negatively. However, the second part of the answer choice goes too far because the authors do not state that there is currently too much regulation. B: No. As in answer choice A, this choice goes too far by assuming the authors' stance on whether or not pollution regulation should increase or decrease from current levels. The authors do not state an opinion either way; therefore, this answer assumes too much. C: No. This is a reversal. In paragraph 1, the authors write, "controlling pollution is a significant economic cost for many firms today." This implies that companies are actually paying more for waste management since the 1960s, not less. D: Yes. In paragraph 1 the authors write, "polluters have little or no economic incentive to stop polluting." This implies that the authors believe that without regulation, a person or company will continue to pollute. Q7 B. This is a New Information question. A: No. While the authors state that such a balance is important for the government to ensure, it is at the national level that the authors believe this regulation should occur, not the state level. B: Yes. This answer choice addresses the water pollution issue in terms of the authors' argument that the balance between economic and environmental concerns should be dealt with on a national level. C: No. The authors speak of consumers paying taxes to pay for pollution control (paragraph 4), but never discuss taxing the polluters themselves. Furthermore, the authors discuss pollution control and "minimum environmental protection standards" (paragraph 5, for example) as the correct approach, not cleaning up pollution after it has occurred. D: No. While the authors don't think state governments should be the ones to impose or enforce regulations (paragraph 5), the authors do think that action should be taken by the national government. A "do nothing" policy on the part of the state government doesn't capture the authors' argument that someone, the national government, should do something

When the facts are in dispute—when there is reasonable doubt that defendants committed the crime with which they are charged—a jury trial is the most appropriate means of adjudication. Whether defendants choose to go to trial depends on their (and their attorneys') assessments of the risks that may be involved. Although the potential gain of complete acquittal is large, the price of defeat may be just as large. If defendants are convicted of the highest charge against them, or if the judge is in the habit of imposing stiffer sentences on defendants who insist on a trial than on defendants who plead guilty to the same offense, conviction at trial may bring a long prison sentence. Certainly it is essential that defendants be free to exercise their rights to a jury trial. A number of legal scholars argue that plea bargaining denies defendants that freedom by penalizing them for exercising it. When defendants convicted at a jury trial receive stiffer sentences than they would have drawn had they pleaded guilty, they are paying a price for having gone to trial; in effect, the state is punishing them for exercising their constitutional rights. Another concern regards procedural rights granted to defendants. The most popular explanation for the courts' ineffectiveness in "putting away" criminals is that the Warren Court put too much emphasis on the rights of defendants. This concern is not new, however. "Our dangers do not lie in too little tenderness to the accused," Judge Learned Hand wrote in 1923. "Our procedure has been always haunted by the ghost of the innocent man convicted. It is an unreal dream. What we need to fear is the archaic formalism and watery sentiment that obstructs, delays, and defeats the prosecution of crime." However, arguments of this sort are rooted in ideological preferences rather than in empirical research. When the data are assembled and analyzed, it becomes clear that the pendulum has not swung too far. Only a handful of criminals go free or escape punishment because of exclusionary rules, search-and-seizure laws, appeals, and other "technicalities" designed to protect defendants' rights. It is only in cases of so-called "victimless crimes" that any significant number of seemingly guilty offenders go free because tainted evidence—evidence acquired as a result of an illegal search, seizure, or arrest—is excluded from court; and even here, the number is considerably smaller than critics assume. Search-and-seizure problems play a larger role in drug cases, especially those where the defendant is charged with possession of a drug. Large numbers of drug arrests are made; in a Rand Corporation study of the prosecution of adult felons in Los Angeles, more than a third of the felony arrests were on drug charges, most of them for possession of marijuana, heroin, and other narcotics and dangerous drugs ("uppers" and "downers"). The prosecutor rejected over 40 percent of the arrests, either through outright dismissal or reduction to a misdemeanor. In the case of arrests for possession of dangerous drugs, most of the rejections were due to insufficient evidence (32 percent) or because they were "trivial offenses." Search-and-seizure problems, illegal arrests, and other violations of defendants' rights accounted for only 17.5 percent of the overall rejections and 25 percent of outright dismissals. When charges against an arrested felon are dropped or reduced to misdemeanors, it usually is not because prosecutors and judges are unduly lenient nor because their heavy caseloads force them to give away the courthouse. Nor, as we have seen, is it because the exclusionary rule "punishes the constable by letting the criminal go free." For the most part, prosecutors drop felony charges or reduce them to misdemeanors because they doubt the defendant's guilt, because they lack the evidence needed to prove guilt, or because they feel the crime was not serious enough, or the defendant not sufficiently culpable, to warrant the stigma and punishment that a felony conviction would bring. Adapted from L.G. Brewster, The Public Agenda. © 1984 by St. Martin's Press. Question 1 Judge Learned Hand's description of "archaic formalism and watery sentiment" (paragraph 3) most likely refers to: A. his concern with the behavior of the Warren Court, which ultimately put too much emphasis on defendants' rights. B. the belief that search and seizure laws lead to guilty people being set free. C. his belief that strict adherence to procedure and fear of punishing innocents impairs criminal prosecution. D. a respected judge's belief that older methodologies and structures are best suited to the application of contemporary justice. Question 3 Based on information in the passage, the author would likely most support which of the following? A. The collection of anecdotes related to victims' rights groups' assertion that criminal defendants are treated too leniently B. Conducting a study to determine whether defendants losing cases that go to trial are subject to longer sentences than those who plea bargain C. Further research into the role defense attorneys play in defending those accused of committing a victimless crime D. The collection of incarcerated criminals' testimonies to determine whether they are aware of the legal consequences of their actions Question 5 As discussed in the passage, the term "exclusionary rules" (paragraph 4) refers to: A. the exclusion of empirical research from arguments rooted in ideological preferences. B. the exclusion of witness testimony from prosecution of victimless crimes. C. the decision of prosecutors not to pursue a case because of insufficient evidence. D. the exclusion of evidence from consideration in a trial because the evidence is illegally obtained. Question 6 Based on information in the passage, the author would be most likely to agree with which of the following statements? A. Prosecutors' ability to drop or reduce felony charges is a fundamental problem with the legal system today. B. Many legal scholars believe criminal defendants' rights are unfairly curtailed by the plea bargaining system. C. The work of police officers, prosecutors, and judges is undermined by criminals who escape punishment due to "technicalities." D. Problems associated with search-and-seizure play a significant role in drug cases brought forth by prosecutors.

Question 1 C. This is an Inference question. A: No. Judge Hand is used as an example to support the assertion that complaints about the Warren Court's defense of criminals have historical precedent; the author states in the second sentence of paragraph 3 that "This concern is not new, however." Therefore, one can also infer that Judge Hand preceded the Warren Court. B: No. As in choice A, the reference to Judge Hand is given as evidence that the concern with putting too much weight on defendants' rights is not new. There is no evidence that Hand is referring to search and seizure laws in particular, however, or even that these specific laws existed in Hand's time (in 1923). C: Yes. Judge Hand states that "Our procedure has been always haunted by the ghost of the innocent man convicted," and these concerns "defeat the prosecution of crime" (paragraph 3). D: No. Judge Hand does not describe or refer to another judge's beliefs with regard to older procedures for the application of justice. If you take "a respected judge" to be Hand himself, note that Hand states that "Our procedure has been always haunted by the ghost of the innocent man convicted," indicating that there is no different older methodology being recommended, but rather a change towards a new stricter methodology. Question 3 B. This is an Inference question. A: No. The author contends that criminal defendants are not in fact treated too leniently. Therefore, there is no evidence from the passage that the author would support the collection of stories illustrating that they are. B: Yes. In the first paragraph, the author indicates that defendants who choose to go to trial rather than pleading guilty may be at risk of getting a longer sentence. In the second paragraph, the author states "it is essential that defendants be free to exercise their rights to a jury trial," and that some scholars argue that defendants who go to trial rather than taking a plea bargain might be penalized for that choice, constituting punishment for exercising constitutional rights. Therefore, we can infer that the author would support research into whether or not this is true. C: No. The author does not discuss the role defense attorneys play in victimless crimes. If it is not an issue in the passage, we cannot infer that the author would like to see further research on the subject. D: No. As in choice C, this issue (criminals' awareness of possible consequences) does not come up in the passage. Therefore, we can't infer that the author would support the collection of information on this subject. Question 5 D. This is an Inference question. A: No. This choice may sound familiar because it uses phrases from the beginning of paragraph 4. However, this is the right answer to the wrong question; in the rest of the paragraph and the passage, the author is discussing exclusion of evidence from trials, rather than exclusion of empirical evidence from ideological arguments. B: No. This choice is too specific to certain kinds of evidence (witness testimony) and certain kinds of trials (prosecution of victimless crimes). C: No. The author draws a distinction in the last two paragraphs between prosecutions that are dropped because evidence was illegally obtained (and therefore excluded) and prosecutions that were dropped because of insufficient evidence. D: Yes. In paragraph 5 the author refers to "tainted evidence" being excluded from trials because it was acquired "as the result of an illegal search, seizure, or arrest." This refers back to the mention of "exclusionary rules" in paragraph 4. Question 6 B. This is an Inference question. A: No. The author does discuss prosecutors' dropping or reducing felony charges in the middle of the seventh paragraph, but the author does not indicate that this is a problem, or an example of prosecutorial leniency. The author suggests that the prosecutors are justified in these actions because they doubt the guilt of the defendant, lack evidence, or believe that the punishment would not match the severity of the crime. B: Yes. This is a restatement of the first two sentences of the second paragraph. The author states that a "number of legal scholars" believe that the plea bargaining system restricts defendants' freedom by punishing them for exercising their right to a trial. C: No. The author believes that "only a handful of criminals go free or escape punishment" because of "technicalities" (paragraph 4). Note that this choice is also inconsistent with the bottom line or primary purpose of the passage. D: No. While the author states in the beginning of paragraph 6 that search and seizure problems play a larger role in drug cases than in other types of cases, he goes on to argue that "search and seizure problems" and "other violations of defendants' rights accounted for only 17.5 percent of the overall rejections and 25 percent of outright dismissals." The use of the word "only" in particular indicates that the author believes that this is not a significant percentage.

Arbitration

dispute resolution

Let's consider the meanings of the words "good" and "bad" as they are used in characterizing ethical conduct. Observation shows that we apply them to acts based on whether or not those acts efficiently achieve their ends. This is not a simple judgment to make. The entanglement of social relations is such that men's actions often simultaneously affect the welfare of self, of offspring, and of fellow citizens. This often results in confusion in judging of actions as good or bad; since actions well fitted to achieve ends of one order may prevent ends of other orders from being achieved. Nevertheless, if we leave aside the issue of conflicting outcomes on these different levels and consider each type of end separately, it becomes clear that the conduct which achieves its goal is regarded as relatively good, and is regarded as relatively bad if it fails. Take for example actions taken in self-interest. A man who fights is said to make a good defense if his defense is well adapted for self-preservation; an unfavorable verdict is given if his actions are futile in serving this purpose. That is, his conduct is right or wrong according to the extent to which it furthers the end of self-preservation. The primary purpose of the author of the passage is to: A. argue that ethical judgments are complex and often confusing. B. argue that an action can be judged by how well it achieves the goal of the action. C. defend one's right to prioritize self-interest over the welfare of others. D. support the claim that self-defense is ethically good if actions taken in self-defense are effective.

B. This is a Primary Purpose question. A: No. This choice is too narrow to be the purpose of the entire passage. While the author does indicate this in the middle of the passage, he goes on to explain a way of clearly judging an action, at least on one level (individual self-interest), based on whether or not the ends of that action are achieved. B: Yes. In the beginning of the passage the author introduces the idea of characterizing or judging the ethical nature of conduct, and then states: "Observation shows that we apply [the words good and bad] to acts based on whether or not those acts efficiently achieve their ends." The example of self-defense supports this claim. C: No. The author admits that self-interest may conflict with the interest of others, and he does not argue that self-interest should generally take precedence. Note that the author states as a preamble to the example of self-defense that he is leaving aside the issue of conflicting ends, and is examining how we would judge individual actions only considering individual goals. D: No. This choice is too narrow to be the point of the entire passage. Note the words "Take for example" that lead into the discussion of self-defense. That phrase tells you that what follows is support for a larger claim.

While the cultural phenomenon known as the "Mommy Wars" is not the result of the internet, the internet has taken what was once the private griping by mothers about the decisions of other mothers, and made that griping extremely public and heated. On virtually any website dedicated to parenting issues, and in the comment forums on any journalistic article that so much as mentions parent-child relationships, one can find the common polarized debates on all Mommy Wars topics, from nursing to sleep training to attachment parenting to the most heated debate among them: vaccination. The most damaging effect of the Mommy Wars is that they have contributed to the Supermom phenomenon, in which mothers are expected to do it all, and do everything right, while navigating a seemingly endless array of binary decisions, the outcome of any of which could, according to those invested in the Mommy Wars, permanently affect a child positively or negatively. Consequently, the mother who has to return to work after six weeks of maternity leave, leaving her baby to be bottle-fed by a sitter, feels endless guilt about abandoning her baby and is made to feel worse by "breast is best" proponents who are ready to divulge the potential harms of baby formula to anyone who will listen. According to the passage, all of the following are true EXCEPT: A. The internet has contributed to the pervasiveness of the Mommy Wars. B. Mothers who return to work six weeks following the birth of their baby should feel guilty about doing so. C. Vaccination is the Mommy Wars topic that generates the most heated arguments. D. Mommy Wars debates give mothers the choice of only two sides of any issue.

A: No. This idea is clearly expressed in the second sentence of the passage. The author states that while the phenomenon did not originate with the internet, the internet has made the debates "public and heated." B: Yes. While the author states that a consequence of the Mommy Wars is that a mother who returns to work "feels guilty," the author herself does not say that such a mother "should" feel guilty. The author indicates that this guilt is part of the "damaging effect" of the Mommy Wars. Therefore the negative tone of the passage ("damaging") is inconsistent with this answer choice. Be careful to separate what the author simply discusses from what the author indicates that she agrees with or believes in. Since this is an EXCEPT question, this is the credited response, because it is the only answer choice among the ones given that is not directly indicated in the passage. C: No. The author states that "one can find the common polarized debates on all Mommy Wars topics, from nursing to sleep training to attachment parenting to the most heated debate among them: vaccination." D: No. The author twice indicates this by calling the debates "polarized" in the first paragraph and noting the "binary decisions" that a mother faces because of the Mommy Wars. Both of these words indicate the presence of only two opposing sides, thereby justifying this answer choice as being present in the passage material.

The Dickey-Wicker amendment, the name of a rider attached to a 1996 appropriations bill for the Department of Health and Human Services, was passed on January 26, 1996 and has had a crippling effect on stem cell research in the United States ever since. This rider prevents the use of any federal funds to support the generation of human embryonic stems cells and the use of human embryos for any research purposes whatsoever. The first human embryonic stems cells were isolated on November 6, 1998 by Dr. James Thompson, using only private funding. Many more human embryonic cells lines were derived subsequently, and though the federal government does not support such research, it continues in the private sector, nonetheless. The major concern is that, while federally funded research on human embryonic stem cells can be tightly controlled, privately funded research is subject to virtually no oversight. Based on the passage, how would the removal of the Dickey-Wicker amendment from the appropriations bill for the Department of Health and Human Services most likely impact stem cell research? A. It would increase the oversight on stem cell research. B. It would decrease the controversial nature of stem cell research. C. It would have neither a positive or negative impact on stem cell research. D. It would increase the amount of privately funded research on stem cells, in an effort to compete with new publicly funded research.

A: Yes. The passage states: "while federally funded research on human embryonic stem cells can be tightly controlled, privately funded research is subject to virtually no oversight." Therefore, the removal of the bill would mean that stem cell research could be federally funded, and federally funded research would most likely have more oversight. B: No. There is no indication in the passage that stem cell research is controversial! Be careful of this "outside knowledge" attractor. Furthermore, there is no evidence that the existence of the amendment itself contributes to controversy over the issue, and thus there is no evidence that removing it would decrease controversy. C: No. The passage states: "while federally funded research on human embryonic stem cells can be tightly controlled, privately funded research is subject to virtually no oversight." Therefore, it is safe to conclude that the removal of the bill would mean an increase in federally funded stem cell research, which would have a positive impact on stem cell research. D: No. There is no indication that privately funded stem cell research would be compelled to compete with federally funded research. This choice is out of scope.

parochial

narrow-minded

It may suffice to state that "Gothic Architecture" is sometimes used to denote in one general term, and to distinguish from the Antique, those peculiar modes or styles in which most of England's ecclesiastical and many of its domestic edifices of the middle ages have been built. It comprehends those styles only in which the pointed arch predominates, and it is then often used to distinguish such from the more ancient Anglo-Saxon and Norman styles. The origin of this kind of architecture can be traced to the classic orders in that state of degeneracy into which they had fallen in the age of Constantine, and afterwards; and as the Romans, on their abandonment of Britain in the fifth century, left many of their temples and public edifices remaining, together with some Christian churches, it was in rude imitation of the Roman structures of the fourth century that the most ancient Anglo-Saxon churches were constructed. This is apparent from an examination and comparison of such with the vestiges of Roman buildings we have existing. Which of the following statements, if true, would most challenge the claims made in the passage? A. A list of English Gothic churches published by a group of renowned architects contains several examples of churches in which rounded arches are most prevalent. B. Most churches created in England since the middle ages have not had the Gothic style of architecture. C. Few churches left standing today exemplify the Roman style. D. Some of the most ancient churches in England have no resemblance whatsoever to fourth-century Roman structures.

A: Yes. The second sentence of the passage states that Gothic "comprehends those styles only in which the pointed arch predominates." The "only" in this statement makes it assertive and definitive. This answer choice suggests that a group of architects agree that some churches with prevalent rounded arches qualify as Gothic. This would undermine the very defining factor, according to the author, of what constitutes a Gothic church. B: No. Note that in the first sentence of the passage, the author states that Gothic architecture is a term applied to "most of England's ecclesiastical and many of its domestic edifices" built in the middle ages. The author does not say anything about the architecture of churches in England following the middle ages. This answer choice is out of scope. C: No. This information does not weaken anything from the passage. The author states that the most ancient Anglo-Saxon churches were constructed in imitation of Roman structures, but says nothing about how many of these are left standing today. Note, too, that the answer choice merely says "few churches left standing" and not even "few ancient churches left standing." That few churches standing today exemplify Roman style has no bearing on the passage. D: No. The author claims that "the most ancient Anglo Saxon churches were constructed" in "rude imitation" of Roman structures, which is not inconsistent with some having no resemblance.

Let's consider the meanings of the words "good" and "bad" as they are used in characterizing ethical conduct. Observation shows that we apply them to acts based on whether or not those acts efficiently achieve their ends. This is not a simple judgment to make. The entanglement of social relations is such that men's actions often simultaneously affect the welfare of self, of offspring, and of fellow citizens. This often results in confusion in judging of actions as good or bad; since actions well fitted to achieve ends of one order may prevent ends of other orders from being achieved. Nevertheless, if we leave aside the issue of conflicting outcomes on these different levels and consider each type of end separately, it becomes clear that the conduct which achieves its goal is regarded as relatively good, and is regarded as relatively bad if it fails. Take for example actions taken in self-interest. A man who fights is said to make a good defense if his defense is well adapted for self-preservation; an unfavorable verdict is given if his actions are futile in serving this purpose. That is, his conduct is right or wrong according to the extent to which it furthers the end of self-preservation. Question 19 The primary purpose of the author of the passage is to: A. argue that ethical judgments are complex and often confusing. Your Answer B. argue that an action can be judged by how well it achieves the goal of the action. C. defend one's right to prioritize self-interest over the welfare of others. D. support the claim that self-defense is ethically good if actions taken in self-defense are effective.

B. This is a Primary Purpose question. A: No. This choice is too narrow to be the purpose of the entire passage. While the author does indicate this in the middle of the passage, he goes on to explain a way of clearly judging an action, at least on one level (individual self-interest), based on whether or not the ends of that action are achieved. B: Yes. In the beginning of the passage the author introduces the idea of characterizing or judging the ethical nature of conduct, and then states: "Observation shows that we apply [the words good and bad] to acts based on whether or not those acts efficiently achieve their ends." The example of self-defense supports this claim. C: No. The author admits that self-interest may conflict with the interest of others, and he does not argue that self-interest should generally take precedence. Note that the author states as a preamble to the example of self-defense that he is leaving aside the issue of conflicting ends, and is examining how we would judge individual actions only considering individual goals. D: No. This choice is too narrow to be the point of the entire passage. Note the words "Take for example" that lead into the discussion of self-defense. That phrase tells you that what follows is support for a larger claim.

Modern forms of communication have created new and exciting ways to disseminate, collect, and use information to identify and solve problems. An internet-based information "feedback loop" that carries information up from the people to relevant institutions and decision-makers and then back down to the citizenry can create targeted "knowledge networks" tapping into specific pools of information. The public-health world in particular could take advantage of these kinds of techniques. Dealing with bioterrorism, in fact, may absolutely require them. In an attack, the millions of Net users could act as sensors, feeding information about illnesses, suspicious activity, and so on to an appointed captain, who would then feed it into the system. Authorities would instantly know what was happening. Experts everywhere — whether a molecular biologist at a university or a grandmother in Dubuque, Iowa, who lived through smallpox—would instantly be tapped, so they could see the information and try to help. Sure, it could be used fraudulently, but the risks would be outweighed by the rewards. In reverse, officials could send the captains instructions on what to tell people to do and real-time information about events. By disseminating reliable, trusted information, the system might prevent panic. Individual Internet users would take the responsibility of passing information to non-Net users. The primary purpose of the passage is to: A. advocate specific policy initiatives to bring about the implementation of an interactive internet-based information collection and dissemination system. B. describe the value of an internet-based information system for preventing panic in the event of a bioterrorism attack. C. discuss the benefits of collecting and distributing information through an internet-based system. D. describing the ways in which modern communication technology can make citizens more active participants in public life.

C. This is a Primary Purpose question. A: No. The author does not offer any specific policy recommendations. For example, he talks about the importance to public health of having an interactive internet information system, but offers no specific ideas on how to bring that about. B: No. This choice is too narrow. The author does describe this, but as an example of the overall value of having an internet-based interactive information system. C: Yes. The author sets out the purpose of the passage in the beginning when he states: "Modern forms of communication have created new and exciting ways to disseminate, collect, and use information to identify and solve problems. An internet-based information 'feedback loop' that carries information up from the people to relevant institutions and decision-makers and then back down to the citizenry, can create targeted 'knowledge networks' tapping into specific pools of information." He then goes on to discuss public health and bioterrorism as an example of the value of such a system. D: No. This choice is too broad. The passage is specifically about an internet-based information and collection system, not about modern communication technology as a whole. The passage is also specifically about information, not about all forms of active participation in public life.

The doubleness of the epigraphs, and the contrast between them—indeed, the contrast which Ivy McKenzie draws between the physician and the naturalist—corresponds to a certain doubleness in me: that I feel myself a naturalist and a physician both; and that I am equally interested in diseases and people; perhaps, too, that I am equally, if inadequately, a theorist and dramatist, am equally drawn to the scientific and the romantic, and continually see both in the human condition, not least in that quintessential human condition of sickness—animals get diseases, but only man falls radically into sickness. My work, my life, is all with the sick—but the sick and their sickness drives me to thoughts which, perhaps, I might otherwise not have. So much so that I am compelled to ask, with Nietzsche: "As for sickness: are we not almost tempted to ask whether we could get along without it?"—and to see the questions it raises as fundamental in nature. Constantly my patients drive me to question, and constantly my questions drive me to patients—thus in the stories or studies which follow there is a continual movement from one to the other. Studies, yes; why stories, or cases? Hippocrates introduced the historical conception of disease, the idea that diseases have a course, from their first intimations to their climax or crisis, and thence to their happy or fatal resolution. Hippocrates thus introduced the case history, a description, or depiction, of the natural history of disease—precisely expressed by the old word "pathology." Such histories are a form of natural history—but they tell us nothing about the individual and his history; they convey nothing of the person, and the experience of the person, as he faces, and struggles to survive, his disease. There is no "subject" in a narrow case history; modern case histories allude to the subject in a cursory phrase ("a trisomic albino female of 21"), which could as well apply to a rat as a human being. To restore the human subject at the centre—the suffering, afflicted, fighting, human subject—we must deepen a case history to a narrative or tale; only then do we have a "who" as well as a "what", a real person, a patient, in relation to disease—in relation to the physical. The patient's essential being is very relevant in the higher reaches of neurology, and in psychology; for here the patient's personhood is essentially involved, and the study of disease and of identity cannot be disjoined. Such disorders, and their depiction and study, indeed entail a new discipline, which we may call the "neurology of identity", for it deals with the neural foundations of the self, the age-old problem of mind and brain. It is possible that there must, of necessity, be a gulf, a gulf of category, between the psychical and the physical; but studies and stories pertaining simultaneously and inseparably to both—and it is these which especially fascinate me, and which (on the whole) I present here—may nonetheless serve to bring them nearer, to bring us to the very intersection of mechanism and life, to the relation of psychological processes to biography. The tradition of richly human clinical tales reached a high point in the nineteenth century, and then declined, with the advent of an impersonal neurological science. Luria wrote: "The power to describe, which was so common to the great nineteenth-century neurologists and psychiatrists, is almost gone now... It must be revived." His own late works, such as The Mind of a Mnemonist and The Man with a Shattered World , are attempts to revive this lost tradition. Thus the case-histories in this book hark back to an ancient tradition: to the nineteenth-century tradition of which Luria speaks; to the tradition of the first medical historian, Hippocrates; and to that universal and prehistorical tradition by which patients have always told their stories to doctors. Q1 Which of the following statements does NOT accurately paraphrase views presented in the passage? A. Patients' feelings are useful in modern case histories. B. The advent of impersonal methodology in modern science caused an abandonment of subjective description. C. The inception of impersonal science did more harm than good in the field of neurology D. Impersonal science has made human subjects peripheral to the study of disease. Q2 Which of the following best describes the author's attitude as expressed in the passage? A. He is fascinated by the relationship between sickness and human experience. B. He is bitter about modern medicine. C. He is eager to make medicine more practical. D. He is critical of the primitive methods of nineteenth-century medicine. Y Q3.By the phrase "only man falls radically into sickness" the author most likely means to indicate that: A. the types of diseases that afflict animals are for the most part different than those suffered by human beings. B. human self-awareness is sometimes undesirable, as instead of just feeling ill, people also may be emotionally traumatized by their illness. C. human patients, unlike animals, can participate in their cure by asking questions of the doctor. D. the intellectual component of illness is part of being human.

Q1.C. This is an Inference/NOT question. A: No. The author continually emphasizes the importance of human feelings in modern case histories (as in paragraphs 3 and 4). B: No. "The advent of impersonal neurological science" can be inferred to have caused the decline of "richly human clinical tales." C: Yes. The author argues that modern neurology would benefit from a return to the older style of including subjective descriptions in the case history, but nowhere does he claim that modern neurology is overall worse off because of the birth of impersonal science. D: No. The author argues that case histories must be deepened in order to "restore the human subject at the centre"; in fact, he points out that the narrow case history "could as well apply to a rat as to a human being." He thus implies that the rise in impersonal science has made human beings peripheral. . This is a Tone/Attitude question. A: Yes. Note the words "interested" and "compelled" (paragraph 1). In other places in the passage as well, he suggests that these issues "drive" him to important "thoughts." B: No. Even though the author is nostalgic about medicine's past and somewhat critical of its present, nothing in the passage shows him to be bitter (this is too extreme). His tone is quite optimistic (see the first paragraph, for instance). C: No. Practicality is not an issue raised in the passage. D: No. The author is nostalgic about, not critical of, nineteenth-century methods. Q3.D. This is an Inference question. A: No. The author does not indicate here (or elsewhere in the passage) that animals and humans get different diseases, but that humans react to the experience of illness differently. B: No. This choice is too extreme. The author does not suggest that we would be better off without the experience of suffering. In fact, he suggests that the experience is a fundamental aspect of our humanity. C: No. This choice brings in words and phrases mentioned in the next paragraph, and is certainly true in reality (and so it "sounds good"), but this is not the issue in phrase cited the question. Animals and humans are never compared in terms of their relationships with doctors and caregivers. D: Yes. The phrase cited in the question stem introduces a major theme of the passage; people's experience of their own illness is an essential aspect of the illness itself, and a fundamental aspect of our humanity.

An unfortunate collision of circumstances—bad example, misleading precedent, high anxiety-producing developments—surrounded the introduction of alcohol to the Lakota by French, British, and American trappers sometime between 1790 and 1830. In the heyday of their independence, most of the tribe's males and some females had traditionally conducted a vision quest, or hanbleceya around the time of the onset of puberty. One by one, they were expected to forsake the safe haven of their families, often for the first time in their lives, and to brave the unknown of open country. Ideally, they remained isolated until, in an induced altered state of consciousness brought on by fear, fasting, thirst, stress, and faith something extraordinary would happen to them, they experienced a flash of insight. This introspection revealed a highly personal mandate for adulthood, an identity, and was an event of unparalleled importance in charting the course of their lives. If, in later years, a reconfirmation was desired, or if circumstances changed, a second hanbleceya might be sought via a sun dance or other ritual. The significance within their culture of the act of vision-seeking predicted to some degree the Lakota's eventual acceptance of alcohol, which seemed to offer a more efficient and less painful route to altered consciousness. Used to seeking and then trusting the validity of a dreamlike state, some Lakota males found the early effects of inebriation appealing. Moreover, they lived in a time that cried out for explicating visions: certainly at no previous period in tribal history had there been a greater need for enabling and enlightening insights in order to understand the direction their world was taking. The literature of frontier transactions is filled with data about the introduction of "spirits" to unsuspecting Native American tribes. European trappers and traders, usually the first members of their societies to reach the interior of the continent, did not exhibit the drinking patterns condoned in London, Paris, or Madrid. The loud, raucous behavior for which they were infamous would not have been tolerated in their home countries. But when Native Americans met such men, they had no way of knowing this, and were inclined to be broad-minded. European traders, after all, were the conduits for manufactured goods—nails, knives, tools, mirrors, cloth—that Native Americans wanted to acquire. A protocol surrounding the trading event developed, which included speeches of welcome and good intentions by tribal leaders, a distribution of food and especially liquor by the traders, an exchange of items, and a final celebration. Many early reports indicate that Native Americans were initially repulsed by the taste of the concoctions ... that the traders insisted they accept. This may have been intensified by a physiological response—prevalent in many East Asian and Native American populations, in which the skin reacts to alcohol with an unpleasant flushing effect—and by the loss of dignity, of control, that drinking engendered. Certainly the brew's taste was as new to North America as the social rules governing its use-rules whose only available guide was the group of rowdy foreigners producing and distributing the often bootleg hootch. Tribe after tribe learned not only to drink, but how to drink from the traders and trappers ... and eventually this "trapper style" of liquor consumption became incorporated into some Indian communities. To the more sedate missionaries, homesteaders, and merchants who arrived later, who had never met a white mountain man but who carried as part of their own cultural baggage a bevy of negative rumors, stereotypes, and fears about "savage redskins," the fact that Native Americans used alcohol at all and the manner in which they seemed to react to it confirmed every horrified suspicion. An accident of circumstance thus over time hardened into self-fulfilling prophesy. Drinking in sprees, drinking everything in sight when it was available (with intermittent periods of abstinence), drinking until drunkenness turned into unconsciousness—all part and parcel of the European-introduced trading ritual—became the hallmark of popular American folk belief about the way Native Americans "naturally'' responded to alcohol. Q1. Which of the following statements about the effects of alcohol on Native Americans CANNOT be inferred from the passage? A. It produced a dreamlike state. B. It gave them visions which helped guide them through difficult periods. C. It caused some unpleasant physical sensations. D. It led them to behave in ways they did not appreciate. How well supported is the author's claim that the Lakota first encountered alcohol between 1790 and 1830? A. Poorly: no direct documentation is offered in support of the claim. B. Strongly: it is supported by documents cited later in the passage. C. Poorly: it is contradicted by first-hand accounts cited elsewhere in the passage. D. Strongly: it is confirmed by the author's later description of the gradual acceptance of alcohol by the Lakota.

Q2. B. This is an Inference/NOT question. A: No. The "dreamlike state" aspect is supported in the second paragraph. B: Yes. The author implies that the Lakota sought guidance from alcohol (at least early on), but not that they found any (don't misconstrue the information in paragraph 2). C: No. This point is made clearly in the fourth paragraph. D: No. This is a reasonable interpretation of the reference to "the loss of dignity [and] control" (paragraph 4) which initially repulsed the Indians. Q1 A. This is an Evaluate question. A: Yes. The date of this event is mentioned in the first sentence, but no specific evidence is cited. B: No. Note that this is the opposite of choice A. No evidence is cited in the passage to support this particular claim. C: No. While paragraph 4 does suggest the existence of first hand accouints, nothing in the description of those accounts is inconsistent with the author's claim about when alcohol was introduced. D: No. The author's description of this gradual acceptance tells you nothing about the time during which alcohol was first introduced to the Lakota.

An anonymous narrative of the first voyage of Vasco da Gama to India was written in 1497 under the title "Roteiro da Viagem de Vasco da Gama em MCCCCXCVII." Although it is called a roteiro, it is in fact a purely personal and popular account of the voyage, and does not contain either sailing directions or a systematic description of all the ports which were visited, as one might expect in a roteiro. There is no reason to believe that it was written by Vasco da Gama. An officer in such high authority would not be likely to write his narrative anonymously. The faulty and variable orthography of the roteiro also renders improbable the hypothesis that Vasco da Gama was the author. The journal of the first voyage of Columbus contains many allusions to the birds which were seen in the course of it by the great discoverer. In this respect the roteiro of the first voyage of Vasco da Gama resembles Columbus' account. The journal of Columbus is the earliest record of an important voyage of discovery which recognizes natural history as an aid to navigators; the roteiro is the next. Which of the following, if true, would LEAST weaken the author's argument in the passage? A. Vasco da Gama, while highly trained in navigation, had little formal education in other areas, including writing. B. A draft of the roteiro that bears striking resemblance to the printed version, and has da Gama's signature on it, is confirmed by historians as being written in da Gama's hand. C. Recording details of the natural history of foreign lands became increasingly popular in the sixteenth century because it gave navigators further evidence that they were in the right geographical area. D. The roteiro contains references to unusual events in its author's life that are consistent with reputable biographies of da Gama.

A: No. This somewhat weakens the passage information, since it could offer an explanation for "the faulty and variable orthography" in the roteiro if it were actually written by de Gama. Since the author claims that de Gama's authorship was unlikely, this information does not "least weaken." B: No. This answer choice substantially undermines the central claim of the passage by providing evidence that da Gama may well have written the roteiro. C: Yes. For "least weaken" questions, the best answer is the one that either strengthens or has no effect. In the final sentence of the passage, the author says, "The journal of Columbus is the earliest record of an important voyage of discovery which recognizes natural history as an aid to navigators, the roteiro is the next." This answer choice is relevant to this information because it makes a correlation between natural history and navigation, saying that explorers after Columbus and da Gama found natural history useful for navigational purposes. Since the other options for this question all weaken the passage information to some extent, this is the best answer out of the ones provided. D: No. This choice weakens the argument by showing a correspondence between da Gama's biography and the life of the author of the roteiro; the author of the passage argues that da Gama is unlikely to be the author.

There can be little doubt that absolutist theories of government, especially when clothed with a religious sanction which appeals to the beliefs of the people at large, go a long ways towards in protecting the power of an absolute ruler. On the other hand, theories like those of John Locke have a tendency to encourage criticism, and to weaken many of the motives which have usually prevented men from offering resistance to the established government. The practical consequences of Locke's theories, as reproduced and improved on by later writers, would probably be found, if we could trace them, to be represented in no inconsiderable degree in the French and American revolutions which occurred about a century after the publication of Locke's Treatises. The idea that kings have a divine right to misgovern their subjects and the denial that the people are, in the last resort, the supreme arbiters of the fate of their rulers, are dangerous paradoxes. Locke's expression of the spirit of liberty in opposition to such ideas, and his effect on the Whig tradition which incorporated his doctrines, became the common heritage of the English people. The author's attitude towards Locke can best be described as: A. critical B. regretful C. neutral D. appreciative

D. This is a Specific Attitude/Tone question A: No. This is the opposite of the author's attitude towards Locke. The author is critical of "absolutist theories" and of "The idea that kings have a divine right to misgovern their subjects and the denial that the people are, in the last resort, the supreme arbiters of the fate of their rulers." And, the author indicates that Locke's work stands in opposition to such ideas and theories, meaning that the author has a positive attitude towards Locke's ideas and influence. B: No. As with choice A, this answer is negative, while the author's tone towards Locke in the passage is positive. Rather than regretting the impact of Locke's ideas, the author indicates that they have had a positive effect. C: No. The author expresses appreciation of Locke's ideas and influence, and therefore his attitude cannot be described as neutral. In particular, the contrast presented in the passage between Locke's ideas and "The idea that kings have a divine right to misgovern their subjects and the denial that the people are, in the last resort, the supreme arbiters of the fate of their rulers" (described as "dangerous" by the author) indicates that the author feels positively towards Locke and his work. D: Yes. First, in the beginning of the passage the author contrasts "absolutist theories of government" with Locke's ideas. Second, it becomes clear that the author appreciates Locke's influence, and disapproves of these "absolutist theories" when he writes "The idea that kings have a divine right to misgovern their subjects and the denial that the people are, in the last resort, the supreme arbiters of the fate of their rulers, are dangerous paradoxes. Locke's expression of the spirit of liberty in opposition to such ideas, and his effect on the Whig tradition which incorporated his doctrines, became the common heritage of the English people. [emphasis added].

The two main theories of arbitration may be described as judicial and political. One might even go so far as to characterize them by saying that the first is based on how arbitration is supposed to work, while the second is based on how it does in fact work. The judicial theory implies that a "just" solution of the dispute does in fact exist, and that it is the duty of the arbitrator to decide on the principles and the facts involved. The arbitrator sits as a private judge, called upon to determine the legal rights and the economic interests of the parties involved as these rights and interests are demonstrated by the information provided by the parties themselves. The political theory, on the other hand, regards arbitration as an extension of both collective bargaining and, of course, collective coercion. The arbitrator functions as a sensitive instrument of sorts, accurately recording the relative strengths of the parties and making sure that the lion gets his share. The author implies that judicial and political theories may differ in that: I.the first is based on idealism, and the second on pragmatism. II.the first is used for legal disputes and the second for political ones. III.in the first the arbitrator focuses more on facts and principles, and in the second the arbitrator focuses more on the parties. A. I only B. II only C. I and III only D. I, II, and III

I: Yes. Early in the passage the author states that the judicial and political theories could be characterized respectively "by saying that the first is based on how arbitration is supposed to work, while the second is based on how it does in fact work." The idealism versus pragmatism distinction in this answer choice appropriately paraphrases this characterization. II: No. This is an overly literal interpretation of the names of the theories, but this distinction is not supported by any information provided in the passage. The difference between the theories is not based on the types of disputes they are used to resolve, but on the basis of the resolution itself. III: Yes. According to the passage, in the judicial theory, "it is the duty of the arbitrator to decide on the principles and the facts involved." In political theory, "The arbitrator functions as a sensitive instrument of sorts, accurately recording the relative strengths of the parties and making sure that the lion gets his share." This answer option paraphrases this distinction.

Every summer new cinematic box office records surpass those of the previous Christmas season's smash hits, as escalating production costs demand larger and larger opening weekend audiences. The studio publicity machines, aided by film analysts of newspapers, television and radio, bang the drums loudly for the latest big budget releases. At the same time, directors, stars, and starlets parade across the television talk show circuits to insure the investments in their most recent productions and promote the continued prosperity of their careers. Yet almost regardless of a film's artistic merit (a concept which includes performance, direction, and production), it is the film consumers, as average ticket-buyers are known to industry insiders, who determine whether a given production is deemed "successful." Movie lovers cover a wide spectrum: from those who worship and obsess over celebrities, to those who thrill to a good story, and those who spend hours and hours debating the subtleties of cinematic themes, extending even to those whose love for film informs and responds to a larger cultural perspective. This last class of movie-goers, the cultural critics, ought to be distinguished from those who offer daily reviews in the mass media [because] the foundations and motives of film criticism include certain defining elements which set it apart from the narrower activity of popular film analysis. The film reviewer occupies a position central to the ongoing viability of the medium, as an intermediary between an industry primarily concerned with efficient productivity and profitable distribution, and the international public whose interest provides the industry its monetary basis. Given the enormous volume of global cinematic output no single analyst can possibly review every film, and so the process of selecting films for popular commentary is, to some degree, subject to the limits of the industry's marketing system. Reviewers feel an obligation to address the general interests of the viewing public, and so the presence of a highly bankable star in a movie will ensure a certain level of analytical attention, regardless of the perceived quality of the film. Similarly, a movie whose subject matter is directly relevant to a variety of topical concerns will in and of itself generate public excitement, and media reviewers must respect that interest if for no other reason than to maintain their own visibility. The cultural critic, on the other hand, tends to focus on larger social perspectives, and so is less concerned with daily grosses and screen availability and competitive rankings and all the other urgent and momentary matters. Where the daily analyst may limit reviews to questions of relative enjoyability, star power and audience appropriateness, the cultural critic expands the view to embrace historical, psychological, sociopolitical and esthetic issues. As essayist J. Aumont points out, the reviewer "informs and offers a judgment of appreciation,"-thumbs up or thumbs down-while the critic is obliged to impart "the richness of the work to as many people as possible" by "deconstructing the elements pertinent to the work" and illuminating "as many aspects as possible in the commentary." Fulfilling those objectives, Aumont argues, empowers the critic "to offer...an interpretation." Based on the passage, a film buff who declares "Without doubt, the best thing about that movie was the psychological development of character" most likely: A. would not go to a film only to see a favorite actor. B. generally chooses films that receive positive reviews. C. would not assume that a movie that had received few mass media reviews was not worth seeing. D. would enjoy this summer's smash hit. Which of the following statements best expresses an assumption underlying the author's depiction of the daily film analysts' motivation for writing their reviews? A. Maintaining their own high public visibility is the most important factor in reviewers' choice of films. B. Daily analysts do not believe their reviews illuminate core facets of consumer psychology and modern culture by describing the elements that lead to popular success of a movie. C. Cultural criticism can illustrate how core cultural patterns of a society are reproduced in the characters and plot lines of films. D. The more the industry spends on marketing and publicity, the more likely a reviewer is to give a favorable response, in order to maintain status within the industry.

Q1 C. This is a New Information question. A: No. See paragraph 2. The author mentions various filmgoer interests, but does not say that decisions made by an individual filmgoer are based only on one factor. B: No. There is no connection made in the passage between positive reviews and psychological themes or character development. C: Yes. If a person appreciates aspects like psychological development, that indicates that he or she may look beyond celebrity of the actors or good plots. Therefore, this buff may find value in films that are overlooked by reviewers, who tend to focus on the most popular movies (see paragraph 3). D: No. See paragraph 1. The passage implies that the "smash hits" do not focus on "psychological development of characters." Q2 B. This is an Inference question. A: No. This choice is too extreme. While this is listed as a factor, the author does not prioritize it over all other factors. B: Yes. The author contrasts reviewers or "daily analysts'' with cultural critics. The former cover narrower ground, expressing "judgements of appreciation," while the latter delve into the deeper social, psychological and artistic significance and meaning of films. This contrast involves the assumption that reviewers do not seek to analyze or interpret films on these deeper levels. C: No. This choice is not directly relevant to the motivation of reviewers (who are distinguished from cultural critics). D: No. This choice is too strong. While marketing will affect what films a reviewer chooses to review, the author does not imply or assume that these reviews will be favorable.

Enlightenment is man's release from his self-incurred tutelage. Tutelage is man's inability to make use of his understanding without direction from another. Self-incurred is this tutelage when its cause lies not in lack of reason but in lack of resolution and courage to use it without direction from another. "Have courage to use your own reason!" - that is the motto of enlightenment. Laziness and cowardice are the reasons why so great a portion of mankind, after nature has long since discharged them from external direction . . . , nevertheless remains under lifelong tutelage, and why it so easy for others to set themselves up as their guardians. It is so easy not to be of age. If I have a book which understands for me, a pastor who has a conscience for me, a physician who decides my diet and so forth, I need not trouble myself. I need not think, if I can only pay - others will readily undertake the irksome work for me. That the step to competence is held to be very dangerous by the far greater portion of mankind . . . is seen to by those guardians who have so kindly assumed superintendence over them. After the guardians have first made their domestic cattle dumb and have made sure that these placid creatures will not dare take a single step without the harness of the cart to which they are tethered, the guardians then show them the danger which threatens if they try to go alone. Actually, however, this danger is not so great, for by falling a few times they would finally learn to walk alone. But an example of this failure makes them timid and ordinarily frightens them away from all further trials. For any single individual to work himself out of the life under tutelage which has become almost his nature is very difficult. He has come to be fond of this state, and he is for the present really incapable of making use of his reason, for no one has ever let him try it out. Statutes and formulas, those mechanical tools of the rational employment or rather misemployment of his natural gifts, are the fetters of an everlasting tutelage. Whoever throws them off makes only an uncertain leap over the narrowest ditch because he is not accustomed to that kind of free motion. Therefore, there are few who have succeeded by their own exercise of mind both in freeing themselves from incompetence and in achieving a steady pace. But that the public should enlighten itself is more possible; indeed, if only freedom is granted, enlightenment is almost sure to follow. For there will always be some independent thinkers, even among the established guardians of the great masses, who, after throwing off the yoke of tutelage from their own shoulders, will disseminate the spirit of the rational appreciation of both their own worth and every man's vocation for thinking for himself. But be it noted that the public, which has first been brought under this yoke by their guardians, forces the guardians themselves to remain bound - so harmful is it to implant prejudices for they later take vengeance on their cultivators or on their descendants. Thus the public can only slowly attain enlightenment. Perhaps a fall of personal despotism or of avaricious or tyrannical oppression may be accomplished by revolution, but never a true reform in ways of thinking. Rather, new prejudices will serve as well as old ones to harness the great unthinking masses. Q1.Suppose a study finds that whenever a certain television personality recommends a particular book, huge numbers of people who otherwise seldom purchase books buy it and read it. How do the results of this study relate to the discussion in the passage of the relationship of guardians to the majority of people? A. The results of the study support the author's contention that frequently people prefer to rely on external authority in making various decisions rather than on their own faculties or discernment. B. The results of the study support the author's suggestion that guardians who are public authorities can, by modeling, encourage others in their own attempts to employ reason. C. The results of the study support the author's assertion that it can be very difficult to approach enlightenment along a strictly individual path. D. The results of the study exemplify the ways in which new prejudices always take hold of the unthinking masses. Q2.For which of the following does the author NOT provide explicit definition or clarification? A. Enlightenment B. Self-Incurred C. Reason D. Tutelage Q3.Which of the following phrases, in context, best epitomizes the author's attitude towards those guardians who encourage others to continue a self-incurred tutelage? A. "lack of resolution and courage" B. "so kindly assumed superintendence" C. "incapable of making use of his reason" D. "there will always be some independent thinkers" Q4.The author implies that the dangers associated with attaining enlightenment: A. are less significant than many have come to expect. B. originate in an experience of failure. C. cause the cowardice that in turn impedes enlightenment. D. affect individuals acting independently more than the public as a whole. Q5.The relationship of the more independent guardians to the public, as described in the final paragraph, is most analogous to which of the following? A. A politician seeking to change public opinion tries to gather the support of other politicians in order to present a unified front on an important social issue B. A teacher encouraging students to explore more figurative uses of language in writing poetry finds that the students have a great deal of trouble understanding symbols and metaphors C. A parent who in the past has successfully forced a child to follow certain rules finds that the child becomes more and more disobedient as she gets older and gains independence D. A minister preaching a new, more open doctrine faces resistance from the congregation and is pressured to return to more traditional teachings Q6.Which of the following, according to the passage, is most likely to contribute to the general emancipation of some portion of the masses from their self-incurred tutelage? A. The public forcing guardians to relinquish aspects of their previous dominance B. The power of each person to overcome the effects of laziness or cowardice, which previously kept them from enlightenment C. Assistance offered by certain well-intentioned authority figures D. A revolution eliminating certain forms of tyrannical oppression The passage suggests that which of the following commonly work(s) against the widespread attainment of enlightenment? The inherent difficulty of acquiring reason A typical reluctance to undertake arduous tasks The consequences of established prejudices A. I only B. III only C. II and III only D. I, II, and III

Q1.A. This is a New Information question. A: Yes. The anecdote in the question stem parallels the author's discussion in the second paragraph of authority being substituted for individual discernment. Following the recommendation of a television personality as to what book to read would be similar to following the recommendation of a pastor, a physician, or a book. B: No. There is no suggestion that the celebrity in question is encouraging the use of reason. In the context of the passage, the use of reason entails independent thought and decision-making. C: No. While those following the television personality's recommendation may not be taking an individual path, the information in the question stem doesn't illustrate the difficulty of doing so. This choice doesn't focus on the correct issue. D: No. This choice is too extreme; there is no suggestion of "new prejudices" taking hold because people are reading the recommended book. Q2C. This is an Evaluate question. A: No. "Enlightenment" is defined in the first paragraph as "man's release from his self-incurred tutelage." B: No. "Self-incurred" is explained in the first paragraph: "Self-incurred is this tutelage when its cause lies not in lack of reason but in lack of resolution and courage to use it without direction from another." C: Yes. The author suggests in the first paragraph that reason is a central component of enlightenment, but the passage does not explain what "reason" itself is. D: No. In the first paragraph the author states that "Tutelage is man's inability to make use of his understanding without direction from another." Q3B. This is a specific Attitude question. A: No. This choice refers to the masses who have not achieved enlightenment (paragraph 1), rather than to these guardians. B: Yes. The author's attitude is certainly negative towards such guardians. In context, "so kindly assumed superintendence" represents a sarcastic criticism of their motives in preserving their power over others. Even if you aren't sure whether "so kindly" is meant sincerely or sarcastically, this is the only choice that relates to the guardians who encourage self-incurred tutelage. C: No. As in choice A, this refers to unenlightened individuals (paragraph 4). D: No. This choice (paragraph 5) references the exceptionable guardians of whom the author approves: those who "after throwing off the yoke of tutelage from their own shoulders, will disseminate the spirit of the rational appreciation of both their own worth and every man's vocation for thinking for himself." Q4A. This is an Inference question. A: Yes. The question stem relates to the information in the third paragraph, where the author states that the danger is "not so great" as the guardians have taken pains to suggest. B: No. This choice misrepresents the role of failure in the passage. The author argues that the guardians use a threat of failure to keep the people in line, but that the real experience of failure would teach people that the danger is actually not so great (paragraph 3). C: No. The author does not argue that danger causes cowardice. Rather, cowardice causes fearful people to avoid taking supposedly dangerous steps towards enlightenment (paragraph 2). D: No. This choice misrepresents the contrast between individuals and groups found in the last two paragraphs. The author doesn't suggest that these supposed dangers affect individuals more than the public as a whole, but rather that it is easier for the public, with the aid of an enlightened guardian, to follow the path out of tutelage than for an individual to do so on his or her own. An enlightened guardian may help the public manage the fears caused by the apparent danger of following this path, but the author doesn't say that the public is less affected by the danger in the first place. Q5.D. This is an Analogy question. A: No. The more independent guardians in the passage appeal to the public, not to other guardians. B: No. While this choice suggests the difficulty of change, there is no indication in it of active resistance to that change, an important theme in the relevant part of the passage. C: No. This choice describes someone resisting authority and becoming more independent, while the cited part of the passage suggests that the public wants to return to "tutelage." D: Yes. The guardians referenced in the question stem are those who seek to pass on enlightenment. According to the author, however, the public, being used to their yokes, "forces the guardians to remain bound." Q6.C. This is a Retrieval question. A: No. This choice relates to the second half of the final paragraph. However, that section of the passage (after the word "but") relates to the public forcing guardians to conform, not to the public forcing guardians to relinquish power. B: No. The author argues in paragraph 4 that it is very difficult for "any single individual to work himself out of the life under tutelage." C: Yes. The question stem points to the beginning of the last paragraph, where the author states that certain guardians might spread the spirit of independent thought. D: No. In the last paragraph the author argues that "true reform in ways of thinking" can never be accomplished through revolution Q7 C. This is an Inference question. I: False. While the author argues that people often lack the courage to use their own reason (paragraph 1), he does not suggest that the attainment or acquisition of reason is inherently difficult. II: True. This statement is supported by the second paragraph's description of the willingness of many to give to someone else the difficult task of reasoning: "I need not think, if I can only pay - others will readily undertake the irksome work for me." III: True. This statement matches the author's discussion in the final paragraph of the influence of prejudices: "Rather, new prejudices will serve as well as old ones to harness the great unthinking masses.

Reconciliation

agreement after a quarrel

Daniel Balderston's assertion that the critics of Jorge Louis Borges have been remiss in their treatment of historical references in Borges' literature, especially those references contemporary to Borges, seems to imply that in-depth historical analysis yields further insight into Borges' works. Balderston suggests that there may be subtleties in Borges' work not evident without a profound historical understanding. However, some of the similarities Balderston has mentioned between real and fictional characters are weak at best, and many would exist by the situation's rationale regardless of authorial intent. Also, even without a historical understanding, subtleties are quite often more obvious than Balderston admits. The word "remiss," in the context of the passage, most nearly means: A. careless. B. overly critical. C. incorrect. D. meticulous.

A: No. Although "careless" is a synonym for "remiss," it does not fit in the context of the passage. Balderston's position is "in-depth historical analysis yields further [emphasis added] insight into Borges' works," and that "a profound historical understanding" may be needed to understand some subtleties in the works. Balderston's implication, then, is that these remiss critics have not delved deeply enough into historical elements in Borges' work. This does not mean that they have not been careful enough, but rather that they either have not understood the need to study this aspect in depth, or that they have failed to go far enough into it. B: No. If anything, Balderston suggests that the critics have not been critical enough since they have not been attentive enough to applying historical analysis to the works of Borges. C: Yes. Balderston's position in reaction to these "remiss" critics is that "in-depth historical analysis yields further insight into Borges' works, and that "a profound historical understanding" may be needed to understand some subtleties in the works. Balderston's implication, then, is that these "remiss" critics have not correctly evaluated the importance of the historical elements in Borges' work; that is, that they have incorrectly treated Borges' historical references. D: No."Meticulous" means characterized by extreme care in the consideration or treatment of details. This is the opposite of the meaning of the word "remiss" in the context of the passage.

It is imperative that post-secondary institutions have in place unambiguous policies concerning cheating and plagiarism that give instructors clear parameters on how to respond to cases they discover among their students. The policy should therefore be consistent across the school and not dependent on individual instructors, who can vary greatly in their approaches to grading and discipline. Our recommendation is that for any case of cheating or plagiarism, a mark of 0 be given on the assignment in question, and that the case be recorded and sent to the appropriate department and sector dean along with evidence supporting the claim. Reporting plagiarism not only protects the instructor, who then has recourse if the student decides to challenge the mark, but also helps institutions keep track of the extent of the problem of intellectual dishonesty, as well as to put policies in place to punish repeat offenders. The main idea of the passage is that: A. an individual instructor must be more consistent in his or her grading and discipline policies. B. cheating and plagiarism are on the rise in post-secondary institutions, so it is more important than ever to have a consistent policy concerning these practices. C. institutions need to have clear regulations about how to respond to plagiarism, including such policies as documentation on the part of the instructor and a failed assignment for the student in question. D. plagiarism is a serious academic offense worthy of garnering a student expulsion from his or her post-secondary institution.

A: No. The author of the passage states that instructors are inconsistent with each other (and that this is why institutions need clear policies), but not that individual instructors are erratic in their own decision-making. B: No. The author mentions nothing about plagiarism being on the rise. This answer choice contains "out of scope" information. C: Yes. This appropriately sums up the main idea of the passage, including the first sentence and the final two sentences. D: No. This answer choice is too extreme. The author makes no mention of expulsion.

It is imperative that post-secondary institutions have in place unambiguous policies concerning cheating and plagiarism that give instructors clear parameters on how to respond to cases they discover among their students. The policy should therefore be consistent across the school and not dependent on individual instructors, who can vary greatly in their approaches to grading and discipline. Our recommendation is that for any case of cheating or plagiarism, a mark of 0 be given on the assignment in question, and that the case be recorded and sent to the appropriate department and sector dean along with evidence supporting the claim. Reporting plagiarism not only protects the instructor, who then has recourse if the student decides to challenge the mark, but also helps institutions keep track of the extent of the problem of intellectual dishonesty, as well as to put policies in place to punish repeat offenders. The main idea of the passage is that: A. an individual instructor must be more consistent in his or her grading and discipline policies. B. cheating and plagiarism are on the rise in post-secondary institutions, so it is more important than ever to have a consistent policy concerning these practices. C. institutions need to have clear regulations about how to respond to plagiarism, including such policies as documentation on the part of the instructor and a failed assignment for the student in question. D. plagiarism is a serious academic offense worthy of garnering a student expulsion from his or her post-secondary institution.

A: No. The author of the passage states that instructors are inconsistent with each other (and that this is why institutions need clear policies), but not that individual instructors are erratic in their own decision-making. B: No. The author mentions nothing about plagiarism being on the rise. This answer choice contains "out of scope" information. C: Yes. This appropriately sums up the main idea of the passage, including the first sentence and the final two sentences. D: No. This answer choice is too extreme. The author makes no mention of expulsion.

. One basic assumption lies at the root of all security analysis: required financial statements, balance sheets, earnings statements, and quarterly reports provide information that is meaningful but potentially misleading. The "Generally Accepted Accounting Principles" (GAAP) offer accountants a wide latitude, with the result that two companies with equivalent production and earnings figures can appear remarkably different based on how each firm chooses to represent itself in statements of earnings and net worth. Some say that it is a waste of time to stumble through the labyrinth of a company's financial statements, since statistics show that a company's past earnings are not predictive of future earnings and, even if they were, an efficient market would already reflect the change in valuation by affecting the price of the company's stock. The author's tone in this passage is best described as: A. confused. B. neutral. C. biased. D. mellifluent.

A: No. There is nothing in the author's tone to suggest he is confused; he talks about information being potentially misleading but not that he has been misled in any way. B: Yes. The author gives no information to suggest he sides with any party on the issue. C. No: See the answer for B above. Simply suggesting that "some" people hold a view does not mean that the author is among them. D. No: This word means "sweet." Avoid choosing words that are unfamiliar to you unless you can definitively rule out the other options.

Knitting and crochet are the yin and yang of the "stick" textile arts. While the layperson might consider the two to be quite similar (after all, both use a stick or two to loop yarn into wearable objects), they are actually as different as night and day. Knitting appeals to the very organized, type-A personality. It is like computer programming - all zeros and ones. Knitting, using two straight, pointed needles, involves only two stitches, the "knit" and the "purl." It is with these two stitches, once mastered, that the knitter, like the computer programmer, can come up with a dizzying array of patterns resulting in an infinite number of objects. This also means that the knitter must master the language of his or her craft. A computer programmer could see 0011010100101 and derive meaning from this. So too, a knitter must be able to decipher the code of a pattern that resembles a long string of commands, such as: CO 15, K1, P2, K3, and so on. Which of the following, if true, would most weaken the author's claim that a knitter must be capable of deciphering a code that resembles a long string of commands? A. Japanese knitting patterns, considered some of the finest in the world, are made up of intricate diagrams instead of coded numbers and letters. B. Some advanced computer programs involve more than just zeros and ones. C. Many who consider themselves to be somewhat disorganized and "scattered" enjoy knitting more than crochet. D. While basic knitting involves just two needles, more advanced techniques include up to six needles, and a plethora of special tools (stitch markers, circular needles, etc).

A: Yes. The author claims at the end of second paragraph that "a knitter must be able to decipher the code of a pattern that resembles a long string of commands, such as: CO 15, K1, P2, K3." This choice suggests that there are alternative types of patterns (those involving diagrams), and therefore undermines the author's claim in the second paragraph. Note that the author uses strong language ("must"). Therefore, one significant case that represents an exception is enough to weaken the claim. B: No. This choice focuses on computer programming, which the author uses merely as an example, and does not focus on the author's claim, which is that "a knitter must be able to decipher the code of a pattern that resembles a long string of commands, such as: CO 15, K1, P2, K3." C: No. This choice focuses on a different claim made in the passage ("Knitting appeals to the very organized, type-A personality"). Note that the question stem asks specifically for information pertaining to the claim about knitting codes; always be attentive to the question stem. A secondary issue is that even if relevant, this statement would not be strong enough to be the correct choice for a "most weaken" question. D: No. This choice focuses on a different claim made by the author: "Knitting [uses] two straight, pointed needles," and does not weaken the claim that "a knitter must be able to decipher the code of a pattern that resembles a long string of commands, such as: CO 15, K1, P2, K3." Note that the question stem asks specifically for information pertaining to that claim at the end of the passage; always be attentive to the question stem.

One basic assumption lies at the root of all security analysis: required financial statements, balance sheets, earnings statements, and quarterly reports provide information that is meaningful but potentially misleading. The "Generally Accepted Accounting Principles" (GAAP) offer accountants a wide latitude, with the result that two companies with equivalent production and earnings figures can appear remarkably different based on how each firm chooses to represent itself in statements of earnings and net worth. Some say that it is a waste of time to stumble through the labyrinth of a company's financial statements, since statistics show that a company's past earnings are not predictive of future earnings and, even if they were, an efficient market would already reflect the change in valuation by affecting the price of the company's stock. The author's tone in this passage is best described as: A. confused. B. neutral. C. biased. D. mellifluent.

A: No. There is nothing in the author's tone to suggest he is confused; he talks about information being potentially misleading but not that he has been misled in any way. B: Yes. The author gives no information to suggest he sides with any party on the issue. C. No: See the answer for B above. Simply suggesting that "some" people hold a view does not mean that the author is among them. D. No: This word means "sweet." Avoid choosing words that are unfamiliar to you unless you can definitively rule out the other options.

Not long ago, I chanced to open a magazine at a story of Italian life which dealt with a curious popular custom. It told of the love of the people for the performances of a strangely clad, periodically appearing old man who was a professional story-teller. This old man repeated whole cycles of myth and serials of popular history, holding his audience-chamber in whatever corner of the open court or square he happened upon, and always surrounded by an eager crowd of listeners. So great was the respect in which the story-teller was held, that any interruption was likely to be resented with violence. As I read of the absorbed silence and the changing expressions of the crowd about the old man, I was suddenly reminded of a company of people I had recently seen. They were gathered in one of the parlours of a women's college, and their serious young faces had, habitually, none of the childlike responsiveness of the Italian populace; they were suggestive, rather, of a daily experience which precluded over-much surprise or curiosity about anything. In the midst of the group stood a frail-looking woman with bright eyes. She was telling a story, a children's story, about a good and a bad little mouse. She had been asked to do that thing, for a purpose, and she did it, therefore. But it was easy to see from the expressions of the listeners how trivial a thing it seemed to them. That was at first. But presently the room grew quieter; and yet quieter. The faces relaxed into amused smiles, sobered in unconscious sympathy, finally broke in ripples of mirth. The story-teller had come to her own. The attitude of the grown-up listeners did but illustrate the general difference between the effect of telling a story and of reading one. Everyone who knows children well has felt the difference. With few exceptions, children listen twice as eagerly to a story told as to one read, and even a "recitation" or a so-called "reading" has not the charm for them that the person wields who can "tell a story." And there are sound reasons for their preference. The great difference, including lesser ones, between telling and reading is that the teller is free; the reader is bound. The book in hand, or the wording of it in mind, binds the reader. The story-teller is bound by nothing; he stands or sits, free to watch his audience, free to follow or lead every changing mood, free to use body, eyes, voice, as aids in expression. Even his mind is unbound, because he lets the story come in the words of the moment, being so full of what he has to say. For this reason, a story told is more spontaneous than one read, however well read. And, consequently, the connection with the audience is closer, more electric, than is possible when the book or its wording intervenes. Beyond this advantage, is the added charm of the personal element in story-telling. When you make a story your own and tell it, the listener gets the story, plus your appreciation of it. It comes to him filtered through your own enjoyment. That is what makes the funny story thrice funnier on the lips of a jolly raconteur than in the pages of a memoir. It is the filter of personality. Everybody has something of the curiosity of the primitive man concerning his neighbour; what another has in his own person felt and done has a special hold on each one of us. The most cultured of audiences will listen to the personal reminiscences of an explorer with a different tingle of interest from that which it feels for a scientific lecture on the results of the exploration. The longing for the personal in experience is a very human longing. And this instinct or longing is especially strong in children. It finds expression in their delight in tales of what father or mother did when they were little, of what happened to grandmother when she went on a journey, and so on, but it also extends to stories which are not in themselves personal: which take their personal savour merely from the fact that they flow from the lips in spontaneous, homely phrases, with an appreciative gusto which suggests participation. Which one of the following best describes the organization and structure of the passage? A. An anecdote is described and followed by an anecdote that contradicts the purpose of the first. A thesis is offered, and reasons for and illustrations of the thesis are provided. B. An anecdote is described and followed by a thesis. Reasons for and illustrations of the thesis are provided. C. An anecdote is described and followed by another anecdote that ultimately agrees with the purpose of the first. A thesis is offered, the reasons for and illustrations of the thesis are provided, and a contrasting thesis is evaluated. D. An anecdote is described and followed by another anecdote that ultimately agrees with the purpose of the first. A thesis is offered, and reasons for and illustrations of the thesis are provided.

A: No. This choice is only partially correct. While the second anecdote at first appears to contrast with the first, it ultimately illustrates the same point: people enjoy storytelling. B: No. This choice is incomplete; the second anecdote is not mentioned. Compare this choice to choice D, which lists all of the major parts of the passage and therefore is the best of the four choices. C: No. This choice is fine up until the last item on the list, but a contrasting thesis is never discussed. D: Yes. This captures the nature of the first three paragraphs — two ultimately related anecdotes about the power of story-telling — and faithfully describes the final paragraph, which states and supports the author's thesis about the power of story-telling.

British, American, and Canadian literature exhibit differing attitudes regarding animals. As evident from works like A. A. Milne's Winnie-the Pooh and Beatrix Potter's books, among them The Tale of Peter Rabbit, the tradition in Britain, even outside of children's literature, has been to depict animals as English gentlemen (and occasionally ladies) in fur. As exemplified by Peter Rabbit, who tends to wear a blue collared shirt and a pair of slippers (which delay his escape from Mr. McGregor's garden when he snags them on the fence), these animals are highly anthropomorphized. In America, literature often involves stories of the hunt, with animals depicted as fierce, mighty, and formidable adversaries. These animals are clearly different in kind rather than degree from humans; even in a work like Faulkner's The Bear, in which the protagonist clearly laments the demise of the title figure, the bear is respected more as a symbol of the wilderness than as a subject with its own life and feelings. Canadian authors attempt to depict the animals as they truly are, and have shown them an unprecedented level of sympathy. While early Canadian literature also has its hunting stories, the animals in them are portrayed as motivated by love and other emotions that were considered in that era to be an exclusively human domain. The existence of which of the following literary works, if representative of animal stories in their own nations, would most weaken the author's description of animal stories in the passage? A. Mary Tourtel's Rupert Bear series, featuring a beloved British comic book figure who is a world-travelling teddy bear who wears a red sweater and a scarf. B. Canadian Charles G. D. Roberts' novel The Heart of the Ancient Wood, in which a bear who has lost her cub to a hunter takes a human girl lovingly under her care and protection; when the girl is older and falls in love with the hunter, the bear tries to kill him in revenge and the girl shoots the bear dead to save her suitor. C. American James Oliver Curwood's novel The Grizzly King, set in 19th century British Columbia, Canada, depicts the hunting of the mighty Thor the bear in order to highlight the need to conserve rather than hunt bear populations. D. American William Kotzwinkle's novel The Bear Went Over the Mountain, in which a bear named Hal Jam finds a screenplay under a tree near a writer's cabin and dons a suit in an attempt to pitch it to Hollywood, with humorous results.

A: No. This is entirely consistent with the examples of Winnie the Pooh and Peter Rabbit as described by the author as typical British literary animals. B: No. While this account appears slightly dissonant with the author's description of Canadian literary animals (since animals "as they truly are" do not normally care for human children), the story nonetheless involves hunting and an animal motivated by love, and is consistent with the passage's description of Canadian literature. C: No. This answer is tempting because it blurs the line between Canadian and American writing, but it still involves hunting a "mighty" animal, a noted mark of American writing, as well as sympathy, a noted Canadian quality. For a "most weaken" question, this is not strong enough. D: Yes. Kotzwinkle's book sounds more like something expected of British literature, since the bear is clearly human-like and in clothing, and not depicted in an adversarial fashion.

Traditional multiculturalists rebel against the prevalent notion that the scholastic canon represents an objective assessment of academic subjects. They argue that the current curriculum bears the image of its creators; that is, it emphasizes the achievements of white Europeans, making only scant reference to other groups. Appropriate curricula, they argue, would emphasize the ability to understand more than one interpretation of data. Conservative multiculturalists assert that traditional multiculturalists intend to remove Western thought and achievement from the curriculum, that their agenda includes a curriculum that is intended to promote white self-recrimination, and that the traditional canon is essentially solid. The author implies all of the following in the passage EXCEPT that: A. the idea that the current school curriculum is an unbiased account of academic subjects is widely accepted as true. B. conservative multiculturalists worry that traditional multiculturalists want to build a curriculum intended to assign blame to white people. C. traditional multiculturalists seek objectivity in the academic canon. D. white Europeans created the school curriculum as it currently exists.

A: No. This is implied in the first sentence of the passage. The author says it's a "prevalent notion" that the scholastic canon represents an objective account of academic subjects; this is the same as saying that the idea is widely accepted as true. B: No. This is implied in the last sentence of the passage; see the reference to "white self-recrimination." C: Yes. While the author says that traditional multiculturalists rebel against the idea that the academic canon is objective, it is not clear whether or not they seek objectivity or even believe objectivity is possible. This is too much of an inferential leap. D: No. This is stated in the passage's second sentence.

If Poliziano's "Favola di Orfeo" was presented in 1472 it must have been written when its author was no more than eighteen years of age. But even at that age he was already famous. He was born in Montepulciano on July 14, 1454. The family name was Ambrogini, but from the Latinized name of his native town turned into Italian he constructed the title of Poliziano, by which he was afterward known. At the age of ten he was sent to Florence, then governed by Lorenzo de Medici. He studied under the famous Greeks Argyropoulos and Kallistos and the equally famous Italians, Landino and Ficino. Gifted with precocious talent, at the age of sixteen he wrote astonishing epigrams in Latin and Greek. At seventeen he translated the "Iliad" into Latin hexameters. His success with this second book attracted the attention of Lorenzo himself; Poliziano was now known as the "Homeric youth" and he was widely hailed the king of Italian scholars and the literary genius of his time. Which of the following pieces of evidence, if discovered, would most undermine the author's argument? A. Some scholars claim that Poliziano was born in 1450. B. Kallistos indicated in his journals that Poliziano was a good but unremarkable student. C. The diaries of Argyropoulos and Ficino indicate that they contributed substantially to Poliziano's written work, and recent scholars corroborate these claims by noting commonalities between Poliziano's writing and that of his teachers. D. Poliziano's birth place is documented to be Florence.

A: No. While this is inconsistent with the author's claim about Poliziano's age, it is not strong enough to significantly undermine it; it only states that "some scholars" make a different claim, not that these scholars are correct. Furthermore, this is a fairly minor point in the passage. B: No. This does not substantially alter passage information. All we know from the passage is that Poliziano was a student of these men sometime as of the age of ten, and that by sixteen he displayed "precocious talent." That one of his several teachers did not find him remarkable at some point between ten and sixteen does not significantly undermine passage information. C: Yes. If Poliziano's teachers had a hand in producing his written work, this would undermine the author's claims of Poliziano's "precocious talent" and greatness, which are central to the passage. D: No. This information is largely irrelevant to the passage information.

Cash-strapped American cities are finding it harder to maintain social services that their citizens and civic leaders alike once took for granted. Population explosions, compounded by the flight of affluent taxpayers to the suburbs, have drained urban resources and forced many cities to make difficult, often painful, choices. Public hospitals, a boon to the poor instituted in great part by Americans and still a distinguishing feature of the American medical industry, are among those institutions under threat of the budgetary ax. The return of tuberculosis and the spread of newer epidemics among urban populations have pushed hospital resources to their limits. Many in city government are looking to another American institution, the free-enterprise system, for succor. The privatization of public hospitals is becoming a popular alternative to propping up struggling taxpayer-supported hospitals. Public hospitals are not the only urban institutions under fire. Schools—long the targets of criticism and in palpable decline in most American cities—also drain as much as a billion dollars from municipal treasuries nationwide. Complete privatization of schools is probably not on the horizon (public schools have been a part of American life since colonial times), but many different uses of government education resources are under serious consideration. The author's tone in this passage can be best described as: A. sensitive to the issues facing municipal governments. B. indifferent to the changes to American public institutions. C. ambivalent about the privatization of American hospitals and schools. D. critical of companies that seek to privatize beloved public institutions.

A: Yes. For example, early in the passage the author states that the drain on urban resources has "forced many cities to make difficult, often painful, choices." The author is therefore aware of, and not unsympathetic regarding, the challenges faced by cities and city governments. B: No. The author expresses some sympathy with the plight of city (municipal) governments faced with difficult choices. Therefore, she cannot be described as indifferent. C: No. Ambivalent means conflicted or torn in two opposing directions; this author does not indicate that she feels strongly one way or the other about privatization, or that she has conflicting feelings. D: No. The author says nothing about companies seeking to privatize institutions, so we cannot infer anything about her feelings about them.

The railways, which would be useless as so much old iron without the teeming population of Europe and its industry, its commerce, and its marts, belong to a few shareholders who are ignorant of the whereabouts of the lines of rails which yield them revenues greater than those of medieval kings. The same is true of the fields, mines, and machines. And if the children of those who perished by thousands while excavating the railway cuttings and tunnels were to assemble one day, crowding in their rags and hunger to demand bread from the shareholders, they would be met with bayonets and grapeshot, to disperse them and safeguard "vested interests." In this monstrous system where the means of production are owned by only a few, the son of the worker on entering life, finds no field which he may till, no machine which he may tend, no mine in which he may dig, without accepting to leave a great part of what he will produce to a master. He must sell his labor for a scant and uncertain wage. His father and his grandfather have toiled to drain this field, to build this mill, to perfect this machine. They gave to the work the full measure of their strength, and what more could they give? But their heir comes into the world with nothing. What is the main idea of the passage? A. Public resources, when held by a few private owners, cease to benefit the public in the ways that they should. B. Nobody should have the right to own soil or other natural resources. C. A revolt is necessary for the public to take resources back from the elite. D. Because soil, mines, coal, and machinery need to be managed by a few individuals, those people should better understand those resources.

A: Yes. The author clearly laments "that all that enables man to produce and to increase his power of production has been seized by the few," and goes on to discuss how this disadvantages the public: soil is not cultivated or is not cultivated with modern methods, and the output of coal is restricted. Although the passage does not directly state that concentration of ownership of machinery restricts production, the context and logical structure of the passage makes this a reasonable inference. Therefore, this choice offers the best summary of the passage's overall point. B: No. This is too extreme. The author doesn't suggest that nobody should own resources; instead, he finds problems with the fact that so many resources are owned by so few people. C: No. This answer is too extreme and out of scope. The author discusses a problem but does not propose any solutions, and never makes mention of a revolt. D: No. This is the opposite of what the author suggests. He indicates that ownership and management by a few is a bad thing, not that it is a necessary thing.

Throughout history parental authority over children has been the norm of western society. The degree and ultimate scope of that power and control, however, has varied over time, with a steady move toward state's rights over those of the family. Today's father can no longer wield ultimate control over his child to the point of legally prescribing life and death as he could under Roman law. He cannot expect full rights to the services and labor of the children regardless of the mother's wishes and the children's best interests as he could under English Common Law. In fact, under modern law, the parent cannot even be assured that he or she will be able to bring the child up in accordance with the practices he or she believes to be dictated by God. However, surely no one would argue that the images from the turn of the century of small children laboring in sweatshops should be revived today. By requiring inoculations, the state has overseen the development of a generation to whom the ravages of smallpox and polio are a mere historical curiosity. Compulsory education has spawned a literacy rate among the highest in the world, and in this arena the state even allows some degree of parental control over the type and content of the schooling. How does the author feel about state authority over children, based on the passage? A. It threatens parental control over children, but the trend has some positive effects as well. B. It must take precedence over parental control when the two are in conflict. C. It was exercised more appropriately in Roman and English Common Law. D. It generally runs counter to a child's best interests.

A: Yes. While the author is clear that former legal systems were too liberal in allowing absolute parental authority, he also expresses concern that under modern law a parent "cannot be assured that he or she" will be able to bring up a child in accordance with what he or she believes is right. The author therefore expresses concern about the increase of the presence of the state in the lives of families, but sees some positive aspects of increased state control as well. B: No. This choice is too extreme. While the author implies that the state does take precedence in some cases, the passage does not support the statement that it must always take precedence. C: No. The author indicates in the middle of the passage that a revival of previous times would not be broadly recognized as a good solution. D: No. The author lists ways in which state control has benefited children: child labor laws, inoculations, and education.

Modern forms of communication have created new and exciting ways to disseminate, collect, and use information to identify and solve problems. An internet-based information "feedback loop" that carries information up from the people to relevant institutions and decision-makers and then back down to the citizenry can create targeted "knowledge networks" tapping into specific pools of information. The public-health world in particular could take advantage of these kinds of techniques. Dealing with bioterrorism, in fact, may absolutely require them. In an attack, the millions of Net users could act as sensors, feeding information about illnesses, suspicious activity, and so on to an appointed captain, who would then feed it into the system. Authorities would instantly know what was happening. Experts everywhere — whether a molecular biologist at a university or a grandmother in Dubuque, Iowa, who lived through smallpox—would instantly be tapped, so they could see the information and try to help. Sure, it could be used fraudulently, but the risks would be outweighed by the rewards. In reverse, officials could send the captains instructions on what to tell people to do and real-time information about events. By disseminating reliable, trusted information, the system might prevent panic. Individual Internet users would take the responsibility of passing information to non-Net users. Which of the following assertions is made in the passage? A. A net-based system can help to prevent bioterrorism attacks. Your Answer B. A net-based system of collecting and disseminating information has some potential flaws. C. Internet users have an advantage over non-internet users in the event of a bioterrorism attack. D. An internet-based interactive information system is required in order to effectively deal with all forms of public-health threats.

B. This is a Retrieval question. A: No. The passage states that such a system can help deal with such attacks, in part through preventing the spread of panic. However, the author does not suggest that it can help prevent the attacks in the first place. B: Yes. The author states that such as system "could be used fraudulently." He does go on to say that "the risks would be outweighed by the rewards," but the risk or flaw still exists. C: No. While this may seem to be true according to common sense, the author does not make this claim. He says that "Individual Internet users would take the responsibility of passing information to non-Net users," and gives no reason to think that this would not happen, or that internet users would in the end have an advantage. D: No. This statement is too broad and too extreme. The author states that "Dealing with bioterrorism, in fact, may absolutely require" such a system, but he does not make this strong of a claim about all public health threats.

The people reject electronic voting. When Ireland embarked on an ambitious e-voting scheme in 2006 that would dispense with "stupid old pencils," as then-prime minister Bertie Ahern put it, in favor of fancy touchscreen voting machines, it seemed that the nation was embracing its technological future. Three years and €51 million later, in April, the government scrapped the entire initiative. High costs were one concern—finishing the project would take another €28 million. But what doomed the effort was a lack of trust: the electorate just didn't like that the machines would record their votes as mere electronic blips, with no tangible record. One doesn't have to be a conspiracy theorist or a Luddite to understand the fallibility of electronic voting machines. As most PC users by now know, computers have bugs, and can be hacked. We take on this security risk in banking, shopping and e-mailing, but the ballot box must be perfectly sealed. At least that's what European voters seem to be saying. Electronic voting machines do not meet this standard. A backlash against e-voting is brewing all over the continent. After almost two years of deliberations, Germany's Supreme Court ruled in March that e-voting was unconstitutional because the average citizen could not be expected to understand the exact steps involved in the recording and tallying of votes. Political scientist Joachim Wiesner and his son Ulrich, a physicist, filed the initial lawsuit and have been instrumental in raising public awareness of the insecurity of electronic voting.... The younger Wiesner said, with some justification, that the Dutch Nedap machines used in Germany are even less secure than mobile phones. The Dutch public-interest group Wij Vertrouwen Stemcomputers Niet (We Do Not Trust Voting Machines) produced a video showing how quickly the Nedap machines could be hacked without voters or election officials being aware (the answer: five minutes). After the clip was broadcast on national television in October 2006, the Netherlands banned all electronic voting machines. Numerous electronic-voting inconsistencies in developing countries, where governments are often all too eager to manipulate votes, have only added to the controversy. After Hugo Chavez won the 2004 election in Venezuela, it came out that the government owned 28 percent of Bizta, the company that manufactured the voting machines. Similarly, the 2004 elections in India were notorious for gangs stuffing electronic ballot boxes in villages. Why are the machines so vulnerable? Each step in the life cycle of a voting machine—from the time it is developed and installed to when the votes are recorded and the data transferred to a central repository for tallying—involves different people gaining access to the machines, often installing new software. It wouldn't be hard for, say, an election official to plant a "Trojan" program on one or many voting machines that would ensure one outcome or another, even before voters arrived at the stations. It would be just as easy to compromise the privacy of voters, identifying who voted for whom. One way to reduce the risk of fraud is to have machines print a paper record of each vote, which voters could then deposit into a conventional ballot box. While this procedure would ensure that each vote can be verified, using paper ballots defeats the purpose of electronic voting in the first place. Using two machines produced by different manufacturers would decrease the risk of a security compromise, but wouldn't eliminate it. A better way is to expose the software behind electronic voting machines to public scrutiny. The root problem of popular electronic machines is that the computer programs that run them are usually closely held trade secrets. (It doesn't help that the software often runs on the Microsoft Windows operating system, which is not the world's most secure.) Having the software closely examined and tested by experts not affiliated with the company would make it easier to close technical loopholes that hackers can exploit. Experience with Web servers has shown that opening software to public scrutiny can uncover potential security breaches. The electronic-voting industry argues that openness would hurt the competitive position of the current market leaders. Some computer scientists have proposed that computer code be disclosed only to a limited group of certified experts. Making such disclosure mandatory for all electronic voting machines would be a good first step for the Obama administration, consistent with his talk about openness in government. Which of the following does the author most explicitly advocate? A. Openness in the government B. A simpler method of recording and tallying votes C. Printing paper votes corresponding to electronic votes D. Allowing a form of public scrutiny of voting software Question 4 With which of the following statements would the author be most likely to agree? A. Voting inconsistencies due to government interference often exist in developed countries. B. Ireland's primary concern with enacting electronic voting was the exorbitant cost. C. Electronic voting machines are generally less secure than mobile phones. D. A potential conflict may exist between increasing voting security and protecting the interests of companies.

Question 4 D. This is an Inference question. A: No. This answer choice is half right/half wrong. Government interference can lead to voting inconsistencies — however, the author refers to this problem in developing, not developed, countries (paragraph 3). There is no evidence in the passage that similar political problems exist in developed countries. B: No. Paragraph 1 states that while "high costs were one concern... what doomed the effort was a lack of trust: the electorate just didn't like that the machines would record their votes as mere electronic blips, with no tangible record." If it was lack of trust that doomed the effort, the primary concern was not with costs. C: No. This choice is too absolute. It implies that all voting machines are less secure than mobile phones, but the passage makes this assertion only about Nedap machines used in Germany (paragraph 2). D: Yes. Openness through "public scrutiny," the author states, is one form of securing e-voting machines (paragraph 5), but the makers of voting machines claim that "openness would hurt the competitive position of the current market leaders" (paragraph 6). Question 5 D. This is an Inference question. A: No. The author favors openness and transparency in relation to voting software. This should not be confused with the last sentence of the passage. The author's views about electronic voting reform happen to coincide with Obama's policies — this does not necessarily mean the author advocates for Obama's ideas about openness in the government in general terms. Note that the question asks what the author most explicitly advocates for. B: No. While the author implies that the complexity of the system was a constitutional issue in Germany (see paragraph 2: "the average citizen could not be expected to understand the exact steps involved in the recording and tallying of votes"), the passage never suggests that the author himself sees complexity as a problem. C: No. The author explains that having a paper ballot in conjunction with electronic voting would allow for voter verification. However, he also mentions that this method would defeat the purpose of having electronic voting (paragraph 5). D: Yes. According to paragraph 6, "Some computer scientists have proposed that computer code be disclosed only to a limited group of certified experts." This would be one (even if limited) form of public scrutiny (see the end of paragraph 5), and the author calls this method a "better way" than either having both paper and electronic ballots or two companies making the machines.


संबंधित स्टडी सेट्स

ABM 1041 Micro Econ - Harvey James

View Set

Ralley 1 - 1+2 circuit rouge - Col - les substantifs (deel 2)

View Set

BrainPop: Charles Darwin & Natural Selection

View Set

Pharm: Cumulative Book Questions Ch. 6

View Set

English - I'm Not Scared by Niccolo Ammaniti

View Set

A&P, Histologie - La science des tissus

View Set